Marys Medicine

 

Webportal.aanem.org

Neuromuscular Update II: Mind the Gap!
Between Theory and Practice
Andrew J. Skalsky, MD
Bassam A. Bassam, MD
Anthony A. Amato, MD
Amanda C. Peltier, MD
Gerald J. Herbison, MD
Björn E. Oskarsson, MD
AANEM 60th Annual Meeting San Antonio, Texas Copyright October 2013 American Association of Neuromuscular & Electrodiagnostic Medicine 2621 Superior Drive NW Rochester, MN 55901 Printed by Johnson Printing Company, Inc.
Please be aware that some of the medical devices or pharmaceuticals discussed in this handout may not be cleared by the FDA or cleared by the FDA for WKHVSHFL¿FXVHGHVFULEHGE WKHDXWKRUVDQGDUH³RIIODEHO´ LHXVHQRWGHVFULEHGRQWKHSURGXFW¶VODEHO ³2IIODEHO´GHYLFHVRUSKDUPDFHXWLFDOVPD EHXVHGLILQWKHMXGJPHQWRIWKHWUHDWLQJSK VLFLDQVXFKXVHLVPHGLFDOO LQGLFDWHGWRWUHDWDSDWLHQW¶VFRQGLWLRQ,QIRUPDWLRQUHJDUGLQJWKH)'$FOHDUDQFHVWDWXVRIDSDUWLFXODUGHYLFHRUSKDUPDFHXWLFDOPD EHREWDLQHGE UHDGLQJWKHSURGXFW¶VSDFNDJHODEHOLQJE FRQWDFWLQJDVDOHVUHSUHVHQWDWLYHRUOHJDOFRXQVHORIWKHPDQXIDFWXUHURIWKHGHYLFHRUSKDUPDFHXWLFDORUE FRQWDFWLQJWKH)'$DW2 Neuromuscular Update II: Mind the Gap!
Between Theory and Practice
Table of Contents
Orthopedic Assessment of the Neuromuscular Patient Andrew J. Skalsky, MD Assistive Devices for the Neuromuscular Patient Reprinted With Permission From Physical Medicine and Rehabilitation Clinics of North America Andrew J. Skalsky, MD Genetics of Peripheral Neuropathies Bassam A. Bassam, MD Anthony A. Amato, MD Amanda C. Peltier, MD Gerald J. Herbison, MD Björn E. Oskarsson, MD resolved according to ACCME standards. Chair: Dianna Quan, MD
and do not necessarily represent those of the AANEM. Objectives - Participants will acquire skills to (1) Explain how to manage orthopedic problems that develop in chronic NM disease, (2) prescribe and
manage assistive devices in NM patients, (3) diagnose genetically mediated nerve and muscle disorders, and (4) quickly assess and answer questions
about common clinical NM scenarios.
Target Audience:
‡ Neurologists, physical medicine and rehabilitation and other physicians interested in neuromuscular and electrodiagnostic medicine‡ Health care professionals involved in the diagnosis and management of patients with neuromuscular diseases‡ Researchers who are actively involved in the neuromuscular and/or electrodiagnostic research Accreditation Statement - The AANEM is accredited by the Accreditation Council for Continuing Medical Education (ACCME) to provide continuing
medical education (CME) for physicians.
CME Credit - The AANEM designates this live activity for a maximum of put in 3.25 AMA PRA Category 1 Credits™. If purchased, the AANEM
designates this enduring material for a maximum of 5.75 AMA PRA Category 1 Credits™. This educational event is approved as an Accredited Group
/HDUQLQJ$FWLYLW XQGHU6HFWLRQRIWKH)UDPHZRUNRI&RQWLQXLQJ3URIHVVLRQDO'HYHORSPHQW &3' RSWLRQVIRUWKH0DLQWHQDQFHRI&HUWL¿FDWLRQ
Program of the Royal College of Physicians and Surgeons of Canada. Physicians should claim on the credit commensurate with the extent of their
participation in the activity. CME for this course is available 10/2013 – 10/2016.
CEUs Credit - The AANEM has designated this live activity for a maximum of 3.25 AANEM CEU's. If purchased, the AANEM designates this
enduring material for a maximum of 5.75 CEU's.
2012-2013 Program Committee
Vincent Tranchitella, MD, Chair Robert W. Irwin, MD David B. Shuster, MD Dayton, OH Thomas Bohr, MD, FRCPC Shawn Jorgensen, MD Zachary Simmons, MD Loma Linda, CA Queensbury, NY Hershey, PA Jasvinder P. Chawla, MBBS, MD, MBA A. Atruro Leis, MD Jeffrey A. Strommen, MD Atlanta, GA Jackson, MS Rochester, MN Maxim Moradian, MD T. Darrell Thomas, MD New Orleans, LA Knoxville, TN 2012-2013 AANEM President
Peter A. Grant, MD Medford, OR Neuromuscular Update II: Mind the Gap!
Between Theory and Practice
Andrew J. Skalsky, MD
Anthony A. Amato, MD
Chief of Pediatric Rehabilitation Medicine, Vice Chairman, Department of Neurology Director, Neuromuscular Division and Clinical Assistant Professor, Department of Pediatrics University of California San Diego 3URIHVVRURI1HXURORJ +DUYDUG0HGLFDO6FKRRODirector, Partners Neuromuscular Medicine Fellowship Program Dr. Skalsky received his medical degree at the University of Minnesota School of Medicine. His internship and residency training was at the University of California (UC)-Davis in Dr. Amato earned his medical degree from the University of physical medicine and rehabilitation, where he also completed a Cincinnati, and completed his residency training at Wilford fellowship in neuromuscular medicine and pediatric rehabilitation. Hall USAF Medical Center in San Antonio, TX, and fellowship Dr. Skalsky is the Chief of Pediatric Rehabilitation Medicine at at The Ohio State University. He is the vice chairman of the Rady Children's Hospital San Diego and an Assistant Professor Department of Neurology and the director of the Neuromuscular in the Department of Pediatrics at the University of California Division and Clinical Neurophysiology Laboratory at Brigham 6DQ 'LHJR +H LV ERDUG FHUWL¿HG LQ SK VLFDO PHGLFLQH DQG and Women's Hospital in Boston. He is also a professor of rehabilitation, neuromuscular medicine, pediatric rehabilitation neurology at Harvard Medical School. He is the chair-elect of the medicine and electrodiagnostic medicine.
Neuromuscular Section of the American Academy of Neurology. He is the director of the Partners (Brigham and Women's Hospital Bassam A. Bassam, MD
/Massachusetts General Hospital) Neuromuscular Medicine Professor of Neurology fellowship program. Dr. Amato is an author or co-author of more Director, Neuromuscular Program and Electromyography than 150 published articles, chapters, and books. He co-wrote the textbook Neuromuscular Disorders with Dr. Jim Russell. He University of South Alabama has been involved in clinical research trials involving patients with amyotrophic lateral sclerosis, peripheral neuropathies, neuromuscular junction disorders, and myopathies, including Dr. Bassam completed residency training in neurology at Wayne trials in patients with dermatomyositis, polymyositis, and State University in Detroit, MI, and neuromuscular disease and inclusion body myositis.
electromyography fellowships at Wayne State University and Mayo Clinic in Rochester, MN. He currently serves as a professor Amanda C. Peltier, MD
of neurology and director of the Neuromuscular Program Assistant Professor, Vanderbilt University Medical Centerand electromyography (EMG) laboratory at the University of 6RXWK $ODEDPD +H LV ERDUG FHUWL¿HG E  WKH $PHULFDQ %RDUGof Psychiatry and Neurology and the American Board of Dr. Peltier earned her medical degree from The Ohio State Electrodiagnostic Medicine and is a diplomat in the subspecialty University, and completed her neurology residency and a three- of neuromuscular medicine. Dr. Bassam has served on various year fellowship in neuromuscular disease, clinical research and American Association of Neuromuscular & Electrodiagnostic EMG at the University of Michigan. She also received a master's Medicine committees as a member, committee chairperson, or degree in clinical research design and statistical analysis from board examiner. His academic interests and achievements are Michigan. Dr. Peltier is an assistant professor at Vanderbilt focused on neuromuscular disorders and EMG.
University Medical Center. Dr. Peltier is an author or co-author RQ  SHHUUHYLHZHG DUWLFOHV VL[ UHYLHZ DUWLFOHV DQG ¿YH ERRNchapters. Dr. Peltier specializes in neuromuscular disorders, with a focus on peripheral neuropathy and autonomic disorders, and neurologic complications of diabetes.
Neuromuscular Update II: Mind the Gap!
Between Theory and Practice
Gerald J. Herbison, MD
Björn E. Oskarsson, MD
Professor, Department of Rehabilitation Medicine Assistant Professor of Clinical Neurology Jefferson Medical College, Thomas Jefferson University Director, Multidisciplinary ALS Clinic University of California Davis Medical CenterSacramento, CA Dr. Herbison earned his medical degree from Stritch School of Medicine at Loyola University. He is a professor in the Dr. Oskarsson completed his medical school in Lund, Sweden Department of Rehabilitation Medicine at Jefferson Medical and did both a neurology residency and a neuromuscular College of Thomas Jefferson University in Philadelphia, PA. He fellowship at University of Colorado, Denver. He now serves has been the director of the clinical electrophysiology clinic since as an assistant professor of clinical neurology and the director 1970. He presently teaches electromyography two and a half days of the Multidisciplinary ALS clinic at University of California a week as well as gross anatomy and kinesiology to residents.
Davis Medical Center. His clinical and research interests largely focus on ALS, but also include other neuromuscular diseases, electrodiagnostic medicine and neuromuscular pathology. Currently he is an NCATS K12 scholar conducting a trial of Mexiletine for the treatment of muscle cramps in ALS.
NEUROMUSCULAR UPDATE II: MIND THE GAP! BETWEEN THEORY AND PRACTICE Orthopedic Assessment of the Neuromuscular
Andrew J. Skalsky, MD
Chief of Pediatric Rehabilitation Medicine, Assistant Professor, Department of Pediatrics University of California San Diego Musculoskeletal complications such as limb contractures, Severe spinal deformity in progressive NMD leads to multiple hip dislocation or subluxation, and scoliosis are common in neuromuscular disease (NMD) (Table). They contribute to seating and positioning, as well as pain, and it can completely increased disability due to decreased motor performance, mobility preclude upright sitting in a wheelchair.3 Screening for spinal limitations, reduced functional range of motion (ROM), loss of deformities is important since it has several clinical implications. function for activities of daily living, decreased quality of life, and Unfortunately, spinal deformity is neither preventable nor increased pain. The severity often can be reduced or delayed by responsive to nonsurgical modalities such as bracing. Unlike early detection and intervention. The rate of NMD progression is idiopathic scoliosis, neuromuscular scoliosis almost always also related to the frequency and severity of scoliosis, hip pathology, progresses. Early detection and screening are crucial for proper and limb contractures with more rapidly progressive conditions and ideal management of scoliosis.
resulting in earlier and more severe musculoskeletal complications.6The individual's current function and prognosis also heavily The Adams forward bend test is the primary screening test for impact decisionmaking and treatment options. Bracing, stretching neuromuscular scoliosis. The test can be easily performed in the programs, and surgery have all been utilized in the prophylaxis and clinic and should be performed on all pediatric patients with treatment of musculoskeletal complications of NMD; however, no NMD since they are at high risk. The test is often performed interventions can be initiated without detection.
in the seated position since many NMD patients are unable to stand but it can be performed either standing or sitting. The screening examination is performed by having the patient bend forward Cerebral Myelomening Duchenne Spinal
DV IDU DV SRVVLEOH ZKLOH ÀH[LQJ WKH FHUYLFDO muscular
muscular
and thoracolumbar spine. Some patients will require postural support if they are unable to sit independently. The patient is viewed from behind focusing on the rib cage. The examiner is looking for one side of the rib cage to be *if injured before adolescent growth spurt higher than the other next to the vertebral column. The convex side of the scoliosis is the side with the rib hump (Fig. 1). In $GDSWHG IURP 'ULVFROO 6: 6NLQQHU - 0XVFXORVNHOHWDO FRPSOLFDWLRQV RI obese patients, smaller curves can be missed, especially in the neuromuscular disease in children. Review. Phys Med Rehabil Clin N Am lower lumbosacral spine.





Figure 1. The Adams forward bend test.
with favorable pulmonary volumes. Because of recent advances in If a spine curve is detected or the patient's body habitus precludes pediatric critical care management and postoperative respiratory the test's sensitivity, spinal radiographs should be performed. therapy support, surgical treatment of spine deformity in DMD Anteroposterior (AP) and lateral spinal radiographs with the can be deferred until Cobb angle measurements approach 45-50 patient either sitting or standing, based on the individual's degrees and may be safely tolerated in patients with forced vital IXQFWLRQ DUH JHQHUDOO  VXI¿FLHQW 2Q WKH $3 ¿OP WKH &REE capacity measurements even below 30-40% predicted.9 angle is measured. The Cobb angle, named after the American orthopedic surgeon John Robert Cobb, originally was used to HIP DYSPLASIA, SUBLUXATION, AND
measure coronal plane deformity on AP plane radiographs in WKHFODVVL¿FDWLRQRIVFROLRVLV17KH&REEDQJOHLVGH¿QHGDVWKHangle formed between a line drawn parallel to the end plate of Hip dysplasia, subluxation, and dislocation are orthopedic the superior vertebra and a line drawn parallel to the end plate abnormalities encountered in children who have NMD. Hip of the inferior vertebra which results in the greatest angle. Serial dysplasia refers to a condition of the hip that may be present at measurements should be performed using the same anatomic or shortly after birth with inadequate acetabular formation. Hip landmarks to ensure commensurable measurements (Fig. 2).
dysplasia is most commonly seen in NMD with congenital paresis such as congenital myopathies, congenital muscular dystrophies, and spinal muscular atrophy type 1. Hip subluxation and/or dislocation are almost always associated with a degree of hip dysplasia. The primary physical examination maneuver screening for hip dislocation or subluxation is the Galeazzi (or Allis) sign. 7KHPDQHXYHULVSHUIRUPHGE OD LQJWKHSDWLHQWVXSLQHÀH[LQJthe hips and knees, and examining the knee heights. A positive test is when the knees are not at the same level (Fig. 4). The pathologic side is the one with the lower knee height and subluxation is often associated with decreased hip abduction ROM. In the setting of complete dislocation, hip abduction can be reduced, normal, or excessive.
The predicted severity of a spinal deformity is highly dependent on the remaining growth of the spine. Joseph C. Risser developed a method for measuring the iliac apophysis which became known as the Risser sign (Fig. 3). Using a score of 1-5, it gives a measure RIWKHSURJUHVVLRQRIRVVL¿FDWLRQRIWKHLOLDFDSRSK VLVDVDPDUNHUof skeletal maturity. Grade 1 is given when the iliac apophysis LV FDOFL¿HG OHVV WKDQ  FRUUHVSRQGLQJ WR SUHSXEHUW  RU HDUO  to the stage before or during the adolescent growth spurt. Grade 3 is up to 75% and corresponds to the slowing of growth. If the Galeazzi sign is positive, an AP radiograph of the pelvis Grade 4 is 75-100% and corresponds to an almost cessation of should be obtained. Hip subluxation and hip dislocation typically growth. Grade 5 is when the iliac apophysis is fused to the iliac crest and corresponds to the end of growth.
an AP radiograph. This measures the femoral head's containment within the acetabulum with respect to Perkin's line. Perkin's line Previous guidelines in neuromuscular patients especially those is drawn vertically though the lateral most aspect of the acetabular with Duchenne muscular dystrophy (DMD) recommended roof (Fig. 5). Shenton's line, which is formed by the medial aspect surgical treatment for scoliosis once angular deformity exceeded of the obturator foramen and the medial aspect of the femoral 25 degrees with a Risser Grade 3 or less, citing an optimal match neck, forms an unbroken arc in the normal hip. However, in a between the Cobb angle as predictive of relentless progression dislocated hip, this arc will be discontinuous (Fig. 5). Hip




NEUROMUSCULAR UPDATE II: MIND THE GAP! BETWEEN THEORY AND PRACTICE Weakness and inability to achieve active joint mobilization throughout the full normal ROM is the single most frequent factor FRQWULEXWLQJ WR WKH RFFXUUHQFH RI ¿[HG FRQWUDFWXUHV /HVV WKDQantigravity knee extension strength places an individual at risk for NQHHÀH[LRQFRQWUDFWXUHV7KHSRVLWLRQLQZKLFKDMRLQWLVVWDWLFDOO SRVLWLRQHG LQÀXHQFHV WKH QXPEHUV RI VDUFRPHUHV SUHVHQW LQ DQ  Figure 5. Anteroposterior pelvis radiograph with a normal right hip and
given muscle. A shortened muscle length may result in up to a 40% loss of sarcomeres.8 A statically positioned limb developing ¿EURWLF FKDQJHV ZLWKLQ WKH PXVFOH ZLOO GHYHORS FRQWUDFWXUHformation in the position of immobilization. Contractures rapidly subluxation usually is diagnosed with a hip migration percentage develop in many NMDs after transitioning to a wheelchair.
of greater than 33%, although others may classify subluxation as mild when it exceeds 20%. Hip dislocation is diagnosed when the Assessing muscle weakness and muscle imbalance can help migration percentage is greater than 100% or the femoral head is predict which joints are at greater risk of developing contractures. completely uncovered.4 $OWKRXJKLPEDODQFHEHWZHHQÀH[RUDQGH[WHQVRUPXVFOHJURXSVhas not been shown to be a major factor leading to contracture Many ambulatory patients with NMD utilize hyperlordosis to formation, contractures are frequently observed when major stabilize the pelvis during ambulation. Hyperlordosis functionally muscle imbalance is present. This is due to reduced ROM since SXWV WKH KLS LQWR IRUZDUG ÀH[LRQ 6LPLODUO  SULPDU  ZKHHOFKDLU the range is dominated by the stronger muscle group. In several users who spend most of the time in the seated position can 10'V WKHUH LV PRUH LQYROYHPHQW RI WKH DQNOH GRUVLÀH[RUV WKDQ observed equinus foot deformities.8 or necessary for the mechanical advantages added over traditional gait patterns. A physical examination maneuver is often needed Treatment of foot deformities depends on the patient's age, WR GHWHUPLQH ZKHWKHU WKH K SHUORUGRVLV DQG KLS ÀH[LRQ DUH ÀH[LELOLW RIWKHIRRWERQ GHIRUPLW DQGPXVFOHLPEDODQFH$ biomechanically advantageous or obligatory contracture. The nighttime or fulltime ankle–foot orthosis (AFO) in a neutral ankle Thomas test, a physical examination test named after the British position custom molded to the foot deformity may decrease the orthopedic surgeon Dr. Hugh Owen Thomas, is used to rule out hip tendency toward further development of the deformity. A supple foot can be managed nonoperatively by a solid ankle AFO in the neutral position.
extended. The test is positive if the patient's opposite/contralateral KLSÀH[HVZLWKRXWNQHHH[WHQVLRQ DVLJQRIWLJKWLOLRSVRDV WKH Distal lower limb surgical interventions in ambulatory patients hip abducts during the test (suggests tight tensor fascia latae); or ZLWK VORZO  SURJUHVVLYH 10' DUH RIWHQ XWLOL]HG ,I D ¿[HG knee extension occurs (implies tight rectus femoris). Of note, a hip deformity is present, surgical intervention may be required to obtain a plantigrade foot. The Coleman block test is commonly absent in this period, it may indicate developmental dysplasia of used in cavovarus feet to determine whether the contracture involves the forefoot, hindfoot, or both (Fig. 7).5,7,10 The patient thigh just touches the abdomen to obliterate the lumbar lordosis. stands on a wood block 2.5 cm thick with the heel and lateral The angle between the affected thigh and the examination table freely over the block. This negates the effect the forefoot may KLS ÀH[LRQ OLPLWV DQ  FRPSHQVDWRU  OXPEDU ORUGRVLV ZKLFK FDQ have on the hindfoot stance. The hindfoot should be bearing the patient's full weight. The correction of hindfoot varus with the SDWLHQWVWDQGLQJRQWKHEORFNVXJJHVWVWKHKLQGIRRWLVÀH[LEOHDQGinterventions should be directed to correct the forefoot position. If the hindfoot varus does not correct while standing on the block, surgical correction of both the forefoot and hindfoot are likely needed to obtain a more plantigrade foot. Rarely is tendon achilles lengthening (TAL) needed to correct a cavovarus foot because the FDOFDQHXVLVDOUHDG LQFDOFDQHXVRUGRUVLÀH[LRQ )LJ 5,7,10 TAL may worsen the cavus deformity by tilting the hindfoot into more calcaneus and the forefoot into more equinus to maintain ground contact. A dorsal closing wedge osteotomy of the midfoot at the apex of the cavus foot deformity is often performed in Charcot–Marie–Tooth disease patients (Fig. 9). The osteotomy improves the forefoot position in relation to the hindfoot. This improves weight bearing biomechanics and can reduce pain.5,10


NMDs are associated with a number of orthopedic complications. Careful screening with simple physical examination maneuvers can lead to earlier detection. Earlier detection often results in proactive parent response which may limit the progression of the complications. The Adams forward bend test is the primary physical examination screening for scoliosis. The examination Figure 7. 7KH &ROHPDQ EORFN WHVW $  7KH HTXLQRYDUXV IRRW DQG DQNOH
is used to track the severity and progression while the Risser score is used to estimate remaining spinal growth. The Galeazzi rigid deformity.
and Thomas tests are useful for screening for the presence of hip pathology. The Coleman block test can be helpful for distinguishing between a rigid versus supple deformity.
Cobb JR. Outline for the study of scoliosis. The American Academy of Orthopedic Surgeons Instructional Course Lectures. Vol. 5. Ann Arbor, MI: Edwards; 1948.
&ROHPDQ66&KHVQXW:-$VLPSOHWHVWIRUKLQGIRRWÀH[LELOLW LQWKHFDYRYDUXVfoot. Clin Orthop Relat Res 1977;(123):60-62.
Hart DA, McDonald CM. Spinal deformity in progressive neuromuscular disease: natural history and management. Phys Med Rehabil Clin N Am 1999;9(1):213-232, viii.
Heckman JD. Campbell's operative orthopaedics. J Bone Joint Surg Am 2003;85:1914.
Johnson BM, Child B, Hix J, Mendicino RW, Catanzariti AR. Cavus foot reconstruction in 3 patients with Charcot-Marie-Tooth disease. J Foot Ankle Surg 2009;48(2):116-24. PubMed PMID: 19232961.
Johnson ER, Fowler WM Jr, Lieberman JS. Contractures in neuromuscular disease. Arch Phys Med Rehabil 1992;73:807-810.
Krause FG, Wing KJ, Younger AS. Neuromuscular issues in cavovarus foot. Normal calcaneal inclination angle.
Review. Foot Ankle Clin 2008;13(2):243-258, vi.
Skalsky AJ, McDonald CM. Prevention and management of limb contractures in neuromuscular diseases. Review. Phys Med Rehabil Clin N Am 2012;23(3):675-687. PubMed PMID: 22938881 Takaso M, Nakazawa T, Imura T, et al. Surgical management of severe scoliosis with high risk pulmonary dysfunction in Duchenne muscular dystrophy: patient function, quality of life and satisfaction. Int Orthop 2010;34(5):695-702.
10. Tullis BL, Mendicino RW, Catanzariti AR, Henne TJ. The Cole midfoot osteotomy: a retrospective review of 11 procedures in 8 patients. J Foot Ankle Surg 2004;43(3):160-165.
Figure 9. 7KH GRUVDO ZHGJH RVWHRWRP  LPSURYHV WKH IRUHIRRW HTXLQXV
without worsening the hindfoot calcaneus.
Assistive Devices for the Neuromuscular
Andrew J. Skalsky, MD
Chief of Pediatric Rehabilitation Medicine, Assistant Professor, Department of Pediatrics University of California San Diego While there are no curative therapies currently available for muscles) and evertors (i.e., peroneus longus and brevis muscles). most neuromuscular disorders (NMDs), there are a number of Instability of ankle inversion and eversion may occur before treatments strategies that may maximize function and improve GH¿FLWV RI GRUVLÀH[LRQ DQG SODQWDU ÀH[LRQ EHFRPH DSSDUHQW quality of life. The proper assistive device should be goal and In this situation, there is obvious ankle weakness, and gait function oriented. Comfort, cosmesis, and self image need to be REVHUYDWLRQ VKRZV VLJQL¿FDQW GH¿FLWV 2UWKRWLF PDQDJHPHQW LQ strongly considered to maximize compliance.4 these cases focuses on using the least restrictive orthoses while providing support for the subtalar joint. A full thermoplastic or metal double-upright ankle–foot orthosis (AFO) provides maximal mediolateral support for the ankle. In addition to Many individuals with an NMD will require some form of bracing PDQDJLQJ GRUVLÀH[LRQ ZHDNQHVV DQG PHGLRODWHUDO LQVWDELOLW  or orthotics for their lower extremities.6,7 A variety of materials are available, and selection is important for a successful outcome. weakness that should normalize heel rise and improve velocity It has been demonstrated that there is poor compliance, as low as and step length. A stiff, nonarticulated plastic AFO accomplishes 20%, with orthoses use among individuals with NMDs. Potential these goals. Anterior trim lines, thicker gauge plastic, or carbon contributors to poor compliance may include that the orthoses highlight the disability, are not essential for limited daily walking, are uncomfortable, and are cosmetically unacceptable.7 Despite the frequent use of orthoses in NMDs, there remains little to no evidence in the literature to support their use. No randomized During the early stages of an NMD, the primary goal of orthotic control trial (RCT) has ever shown orthoses to be effective management should be on trying to prevent contracture or LQWHUYHQWLRQV7KHRQO 5&7HYDOXDWLQJWKHHI¿FDF RIGD WLPH deformity and minimizing discomfort. As weakness emerges, RUWKRVHV VKRZHG QR VLJQL¿FDQW LPSURYHPHQW LQ IXQFWLRQ SDLQ orthoses can help compensate for the weakness. During the later quality of life, general health, vitality, or social functioning when stages when the weakness is marked, more substantial orthoses participants were randomized to receive a foot orthosis versus a may be needed to maintain effective ambulation. However, sham insole.2 One RCT evaluated the utility of nighttime splinting to maximize compliance, the orthoses need to be minimally using a preformed splint. The 6-week intervention did not have intrusive, comfortable, and cosmetically acceptable.
D VLJQL¿FDQW HIIHFW RQ SDVVLYH GRUVLÀH[LRQ RU HYHUVLRQ UDQJH RImotion or on muscle strength around the ankle in individuals with NMDs with distal involvement can result in weakness of the Charcot–Marie–Tooth disease type 1A (CMT1A). The authors primary ankle invertors (i.e., tibialis anterior and posterior concluded that wearing night splints does not increase ankle range of movement or strength in people with CMT1A.5 A single subject walker. The one exception is for patient's with ataxic conditions experimental design did demonstrate that optimized orthoses can like Friedreich's ataxia when the ataxia is more disabling than LPSURYH R[ JHQ FRQVXPSWLRQ HI¿FLHQF DQG GHFUHDVH SHUFHLYHG ZHDNQHVV 7KH VWDELOLW RI D IRXUSRLQW ZDONHU FDQ DGG VLJQL¿FDQW aid in balance.
The much more commonly used front wheeled walker (FWW) is slightly less stable than a pick-up walker, but it can be slid on the Canes can be a valuable gait aid to prevent fall and injury. The JURXQG ,W UHTXLUHV VLJQL¿FDQWO OHVV VWUHQJWK WR DGYDQFH D ):: single point cane is the most common cane used today. It is versus a pick-up walker. A platform trough can be added to allow primarily a balance aid and does very little to compensate for the user to weightbear through the elbow and shoulder. This is weakness. Forearm crutches (also known as Canadian crutches especially helpful if the user has distal upper limb weakness.
or loftstrand crutches) can help compensate for lower limb weakness by transferring some of the lower limb weightbearing A four-wheeled walker is much less stable than a FWW but also to the upper limbs. This improves gait stability and endurance. requires less strength to advance; however, a four-wheeled walker )RUHDUP FUXWFKHV FDQ EH ¿W ZLWK D IRUHDUP WURXJK WR DOORZ PRUH can slip out on the user very quickly. For this reason, most four- weightbearing through the elbow and shoulder which is especially wheeled walkers are equipped with hand brakes to reduce this KHOSIXO LI WKH XVHU KDV VLJQL¿FDQW GLVWDO ZHDNQHVV )LJ   $[LOODU risk. Many four-wheeled walkers also come with a fold down crutches or standard crutches are rarely used in NMDs since they seat to allow the user to rest. Four-wheeled walkers are primarily are predominantly helpful for unilateral weakness or non-weight for improved balance and endurance. The use of a four-wheeled bearing as in the case of a fracture.
walker should be limited to patients with only mild weakness due to the inherent instability of the walker.
Bathrooms are the most accident prone location within the home. Many individuals with weakness due to a NMD are safer sitting to bath or shower. If the patient possesses adequate strength to do a stand pivot transfer or short distance ambulation, a shower FKDLU RU WXE EHQFK VKRXOG VXI¿FH )LJ   ,I D SDWLHQW LV QRW DEOHto transfer independently, a roll in shower chair or tub transfer system is needed, depending on the layout of the user's bathroom. If a roll in shower with no entry lip is available, there are a number RI UROOLQJ VKRZHU FKDLUV RQ WKH PDUNHW WKDW ZLOO VXI¿FH ,I WKH XVHUKDV VLJQL¿FDQW ZHDNQHVV DQG FDQ WROHUDWH VLWWLQJ XSULJKW IRU RQO a short time, a tilt-in-space or reclining shower chair may be needed. For the more traditional tub/shower combination, a tub transfer system is needed (Fig. 3). Some systems also support the lower limbs and some are able to tilt in space, which is especially helpful if the user has poor trunk control and strength.
elbow in comparison to standard forearm crutches. Several different types of walkers are available to assist individuals with NMDs. The most stable and supportive walker is the four-point or four-prong walker. It is also known as a pick-up walker; Figure 2. Padded Tub transfer bench
however, this type of walker is rarely used since it requires the user to lift and advance the walker. Most individuals requiring this much support do not have adequate strength to advance the 12 NEUROMUSCULAR UPDATE II: MIND THE GAP! BETWEEN THEORY AND PRACTICE In addition to the useful equipment aids discussed above, there are a number of additional items that can substantially improve a patient's quality of life, including hand-held showers, grab bars, raised toilet seats, commode chairs, and activities of daily living aids (e.g., sock aid, grabbers, etc.).
1. Doyle JF, Grocott-Mason R, Hardman TC, Malik O, Dubrey SW. Midodrine: use and current status in the treatment of hypotension. Br J Cardiol 2012;19:34-37.
2. Iodice V, Kimpinski K, Vernino S, Sandroni P, Fealey RD, Low 3$ (I¿FDF RI LPPXQRWKHUDS LQ VHURSRVLWLYH DQG VHURQHJDWLYHputative autoimmune autonomic ganglionopathy. Neurology 2009;72:2002-2008.
3. Koike, Haruki, Hirohisa Watanabe, and Gen Sobue. The spectrum of immune-mediated autonomic neuropathies: insights from the clinicopathological features. J Neurol Neurosurg Psychiatry 2013:84(1);98-106.
4. Low PA. Autonomic nervous system function. J Clin Neurophysiol to allow the user to slide over the tub for a shower or hand bath.
5. Low PA. Evaluation of sudomotor function. Clin Neurophysiol 6. Low PA. Laboratory evaluation of autonomic function. Advances in Muscle weakness can greatly impact sleep quality. The average Neurophysiol, Suppl to Clin Neurophysiol 2004;57:358-368.
sleeper subtly repositions two to four times per hour during the 7. Novak P. Assessment of sympathetic index from the Valsalva deeper stages of sleep. If an individual is too weak to reposition maneuver. Neurology 2011;76:2010-2016.
independently, several nocturnal awakenings can occur resulting 8. Olney RK. Clinical trials for polyneuropathy: the role of nerve in poor sleep quality. A bed with rails may be all that is needed conduction studies, quantitative sensory testing, and autonomic for a patient to regain the ability to reposition independently. function testing. J Clin Neurophysiol 1998;15:129-137.
$ KRVSLWDO EHG FDQ DOVR SURYLGH WKLV VDPH EHQH¿W LQ DGGLWLRQ WR 9. Parsaik A, Allison TG, Singer W, Sletten DM, Joyner MJ, Benarroch the ability to raise and lower the head and foot of the bed. Low EE, Low PA, Sandroni P. Deconditioning in patients with orthostatic pressure or alternating pressure mattresses or mattress overlays can reduce nocturnal awakening by changing the pressure point 10. Shibata S, Fu Q, Bivens TB, Hastings JL, Wang W, Levine BD. distribution (Fig. 4). Improved sleep for the patient can also mean Short-term exercise training improves the cardiovascular response improved sleep for the patient's caregiver.
to exercise in the postural orthostatic tachycardia syndrome. J Physiology 2012;590:3495-3505.
Figure 4. An alternating pressure mattress overlay pad and pump can
UHGXFHQRFWXUQDODZDNHQLQJ
Genetics of Peripheral Neuropathies
Bassam A. Bassam, MD
Professor of Neurology Director, Neuromuscular Program and Electromyography Laboratory University of South Alabama bilaterally. Muscle strength examination showed mild 4+/5 hand grip and interossei muscle weakness on the Medical Research A 28-year-old male presented with a history of slowly progressive muscle weakness since he was a teenager, affecting the legs more and other upper and lower limb muscle groups. There were no has numbness, tingling, and decreased sensation in the feet and muscle twitches, fasciculations or myotonia observed. Tendon UHÀH[HV ZHUH DEVHQW WKURXJKRXW H[FHSW IRU WUDFH ELFHSV UHÀH[ He denies recurrent attacks of focal weakness or palsy, muscle stiffness, and twitches, or spasms, except for infrequent legs sensation was absent at the toes and decreased at the ankles. cramps. He had normal motor and developmental milestones; Pinprick and touch sensation were diminished distally in all limbs however, he has not been involved in any competitive athletic symmetrically, with a stocking-glove distribution. Joint position activities, and he has a history of frequent tripping and ankle sense was decreased in the toes. Coordination and cerebellar sprains since he was 9 years old. IXQFWLRQWHVWVZHUHDOOQRUPDO+HZDONHGZLWKÀRSS VWHSVDQGwas unable to walk on the heels. Romberg sign was negative.
The patient's past medical history is unremarkable. He does not smoke tobacco, drinks two to three beers on the weekends, and Initial Differential Diagnosis
has no history of elicit drug use. The patient's father is diabetic and has a long standing history of leg weakness and sensory The patient presented with chronic, slowly progressive distal disturbances. The patient has a healthy 25-year-old brother and weakness and sensory loss. The neurological examination 23-year-old sister.
VKRZHG GLVWDO PRWRU DQG VHQVRU  GH¿FLWV DEVHQW UHÀH[HV DQGno symptoms or signs of upper motor neuron or central nervous system (CNS) involvement. Thus chronic motor and sensory peripheral polyneuropathy was the initial clinical diagnosis. The general physical examination was notable for mild trophic skin Distal spinal muscular atrophy or distal muscular dystrophy is changes of the feet and distal legs and high arch feet with normal VSLQH DOLJQPHQW EXW RWKHUZLVH WKH ¿QGLQJV ZHUH QRUPDO 7KH inherited causes of peripheral polyneuropathy are considered. neurological examination revealed normal cognitive function and cranial nerves. Motor examination showed symmetric atrophy of polyradiculoneuropathy (CIDP) is a possibility. Metabolic or toxic anterior leg muscles and subtle atrophy of intrinsic hand muscles causes of peripheral polyneuropathy are not highly suspected since the patient has no history of diabetes mellitus, other metabolic CMT disease is the most common type of IPN, affecting over 2.5 diseases, or exposure to medications or toxins. A hereditary million people worldwide of all races and ethnicities, having an neuropathy, such as Charcot–Marie–Tooth (CMT) disease, was estimated prevalence of 1 in 2,500 persons with over 125,000 suspected in view of a childhood onset and a suspicious family people in the United States affected.1-3 Those affected often show history. The history of neuropathy in the father suggests an similar clinical manifestations despite having several genetically- autosomal dominant disease, although diabetic neuropathy is distinct disorders. The clinical features usually present in also a consideration. An axonal or a demyelinating hereditary childhood or early adulthood with distal, symmetrical weakness neuropathy can only be differentiated reliably by electrodiagnostic (EDX) studies or nerve biopsy. Other inherited neuropathies such of variable severity, and most patients are not severely disabled as familial amyloidosis, juvenile- or adult-onset metachromatic and remain ambulatory throughout life. Occasionally, they are leukodystrophy, Krabbe disease, or adrenomyeloneuropathy are associated with other characteristic features, such as hearing loss, not likely in this case scenario. Familial amyloidosis neuropathy phrenic nerve involvement, or scoliosis.4 XVXDOO  KDV SURPLQHQW DXWRQRPLF GH¿FLW 0HWDFKURPDWLFleukodystrophy, Krabbe disease, and adrenomyeloneuropathy Historically, for nearly 100 years,e IPNs were described as a affect both the CNS and peripheral nervous system (PNS) and single disease entity, collectively called CMT, and named after usually are associated with progressive CNS manifestations and the three neurologists who separately described the disease in the late 1880s. Gilliatt and Thomas, in 1957, showed markedly slowed nerve conduction velocities (NCVs) in one form of CMT.5 Laboratory Tests and Electrodiagnostic
+RZHYHU ' FN DQG /DPEHUW LQ  ¿UVW FODVVL¿HG WKH ,31V into hereditary motor and sensory neuropathy (HMSN) and hereditary sensory and autonomic neuropathy (HSAN) based on Laboratory studies including blood count, serum chemistry, electrophysiological features, mode of inheritance (i.e., autosomal creatine kinase, glucose tolerance test, serum B12 and folate, dominant, autosomal recessive, or X-linked), pathology (i.e., thyroid studies, sedimentation rate, anti-nuclear antibodies, demyelinating or axonal), and the age of onset (i.e., childhood/rheumatoid factor, serum protein and immnunoelectrophoresis, adulthood or infancy).6 Among those, the majority of patients and Sjögren antibodies were all normal.
have autosomal dominant demyelinating neuropathy (CMT1) with upper limb NCVs less than 38 m/s and the autosomal Nerve conduction studies (NCSs) showed increased distal motor dominant axonal neuropathy (CMT2), with the X-linked CMT latencies, prolonged F-wave latencies, uniformly reduced motor the next most common, affecting 10-15% of all CMT patients.7 conduction velocity to less than 60% of normal with no conduction EORFN RU VLJQL¿FDQW GLVSHUVLRQ DEVHQW GLVWDO FRPSRXQG PXVFOH The rapid unprecedented advances in molecular genetics, cell biology, and diagnostic testing over the last 25 years have enabled sensory nerve action potentials (SNAPs) in the lower extremities identifying a growing number of chromosomal locations and and ulnar nerve bilaterally, and reduced SNAPs amplitude of the genetic mutations involving Schwann cell structural proteins, PHGLDQ DQG VXSHU¿FLDO UDGLDO QHUYH 1HHGOH HOHFWURP RJUDSK  transcription factors, axonal transport, mitochondrial function, (EMG) examination revealed distal chronic neurogenic changes protein translation, and others. Diverse pathogenetic mechanisms including reduced recruitment and large motor unit action KDYH EHHQ LGHQWL¿HG LQFOXGLQJ GLIIHUHQW PXWDWLRQV LQ WKH potentials in the feet, anterior leg, and intrinsic hand muscles same gene, duplication, deletion, point mutations, and allelic heterogeneity, leading to further dividing each type into several subtypes. It has been a challenge for physicians to keep pace with these new discoveries. This review provides an overview of a demyelinating motor and sensory peripheral polyneuropathy, most likely an inherited rather than an acquired neuropathy in as well as a logistic approach for genetic testing of various view of the early age of symptoms onset and the uniform and subtypes (Table 1). Additional useful and relevant information symmetrically slowed conduction velocity without conduction and recent CMT mutations database can be found on the website block or dispersion. Therefore, DNA diagnostic testing for demyelinating hereditary motor sensory polyneuropathy was requested, and it revealed peripheral myelin protein-22 (PMP22) gene duplication, chromosome 17p11.2, diagnostic of CMT type 1A (CMT1A).
Autosomal Dominant Demyelinating Charcot–Marie–
HMSN type 1, or CMT1, is the most common form of CMT and accounts for about 50-60% of all cases, with disease onset before Inherited Peripheral Neuropathies
the second decade of life in most cases. The clinical features follow a chronic, slowly progressive classical prototypic course The inherited peripheral neuropathies (IPNs) are a heterogeneous including clumsy or slow running in childhood; distal weakness group of genetic peripheral nerve disorders. They are among the and muscle atrophy which starts in the feet and anterior legs, and most common inherited neurologic disorders, accounting for later affects the hands; distal sensory loss of all sensory modalities nearly 40% of chronic neuropathies, and may account for as many GHVSLWH GLVSDUDWHO  IHZHU VHQVRU  FRPSODLQWV JDLW GLI¿FXOW  NEUROMUSCULAR UPDATE II: MIND THE GAP! BETWEEN THEORY AND PRACTICE Among patients with CMT1 the following subtypes have been PMP22 duplication CMT1A is the most common CMT subtype, accounting for 70- 80% of CMT1 and over 50% of all CMT cases, and in 1989 became WKH¿UVWIRUPPDSSHGXVLQJPROHFXODUWHFKQRORJ 9 The disease presents in early childhood with classic feet deformity, distal uniform slowing NCVs. Nerve biopsy shows demyelination with typical onion bulbs. The disease is caused by duplication of a 1.5 megabases fragment in the short arm of chromosome 17p11.2-12harboring PMP22 in 75-80% of CMT1A cases.10 The duplication may arise as a de novo event, and it accounts for sporadic cases of CMT1A. PMP22 protein is expressed in the compact zone of the myelin sheath, maintaining structural integrity of myelin and accounts for about 5% of myelin sheath protein. A small number of CMT1A cases have PMP22 point mutations, and are usually more severely affected, whereas PMP22 deletion is associated with hereditary neuropathy with liability to pressure palsy (HNPP).11 CMT1B accounts for about 10-20% of CMT1 cases, with clinical presentation either as an early-onset severe motor and sensory neuropathy or late-onset presentation similar to CMT1A. It is caused by mutations in the myelin protein zero (MPZ) gene, mapped to chromosome 1q22-23.12 MPZ is a member of the immunoglobulin family and represents approximately 50% of PNS myelin. It localizes to the compact zone of peripheral nerve myelin and maintain tight links between adjacent myelin layers. Adie's pupil and postural tremor is common in patients with CMT1B, and it was previously described as Roussy–Levy syndrome.13 The remaining CMT1 subtypes are rare. CMT1C is due to a PXWDWLRQ LQ OLSRSRO VDFFKDULGHLQGXFHG WXPRU QHFURVLV IDFWRUĮ (LITAF) mapped to chromosome 16p13-12 expressed on Schwann cells. CMT1D is mapped to chromosome 10q21-22 due to a mutation in early growth response 2 (EGR2) gene, a transcription factor involved in Schwann cell differentiation, which also causes Dejerine–Sottas syndrome (DSS). CMT1E is associated with $' DXWRVRPDO GRPLQDQW $5 DXWRVRPDO UHFHVVLYH &+1 FRQJHQLWDO deafness and is due to point mutations in the PMP22 gene, allelic hypomyelinating neuropathy, CMT=Charcot–Marie–Tooth disease, WR&07$&07)LVGXHWRQHXUR¿ODPHQWOLJKWFKDLQ 1()/ Cx32=Connexin 32, DI=dominant intermediate, DNM2=dynamin gene mutations, chromosome 8p21, and presents in infancy or 2, DSS=Dejerine–Sottas syndrome, EGR2=early growth response early childhood with a clinically heterogeneous demyelinating or *$56 *O F OW51$ axonal CMT phenotype (CMT2E)14,15 (Table 1).
GLIIHUHQWLDWLRQSURWHLQ*-% JDSMXQFWLRQȕ+1$ KHUHGLWDU QHXUDOJLFDP RWURSK  +133 KHUHGLWDU  QHXURSDWK  ZLWK OLDELOLW  WR SUHVVXUH Autosomal Dominant Axonal Charcot–Marie–Tooth
SDOV  +63% KHDW VKRFN SURWHLQ ȕ +63 KHDW VKRFN SURWHLQ  .,)% NLQHVLQ IDPLO  PHPEHU % /,7$)6,03/( OLSRSRO VDFFKDULGH CMT2, also referred to as the "axonal CMT," is less common than LQGXFHG WXPRU QHFURVLV IDFWRUĮ 0)1  PLWRIXVLQ  03= 3 P HOLQ the CMT1 forms, estimated to be about 25% of the dominantly- SURWHLQ ]HUR 07050705 P RWXEXODULQ UHODWHG SURWHLQ  inherited CMT cases. The clinical features are similar to CMT1, 1'5* 1P FGRZQVWUHDPUHJXODWHGJHQH1()/ QHXUR¿ODPHQWOLJKW DQG LW LV RIWHQ GLI¿FXOW WR GLVWLQJXLVK EHWZHHQ WKH WZR ZLWKRXW 303 SHULSKHUDO P HOLQ SURWHLQ 5$% 5DVUHODWHG SURWHLQ 5DE utilizing EDX studies and nerve biopsy. However, generally, 6(37 6HSWLQ  6+7& 6+ GRPDLQ DQG WHWUDWULFRSHSWLGH UHSHDWV CMT2 has a later age of onset, there is less involvement of the FRQWDLQLQJ SURWHLQ  ;' ;OLQNHG GRPLQDQW ;5 ;OLQNHG UHFHVVLYH KDQG PXVFOHV WHQGRQ UHÀH[HV DUH GLPLQLVKHG DQG DEVHQW DW WKH <$56 W URV OW51$V QWKHWDVH ankles, and peripheral nerves are not enlarged. A few CMT2 subtypes may have additional clinical features, including optic high-arched feet or hammer toes in 50-70% of cases.8 NCVs DWURSK GHDIQHVVUHVSLUDWRU GLI¿FXOW GXHWRSKUHQLFQHUYHSDOV  are uniformly slow (i.e., median nerve less than 38 m/s) without YRFDOFRUGSDUDO VLVDQGS UDPLGDOWUDFWVLJQV6HQVRU GH¿FLWRQ conduction block or prominent dispersion. Nerve biopsy shows neurological examination and EDX studies are prominent despite demyelination, onion bulb formation, and variable axonal loss. WKH ODFN RI VLJQL¿FDQW VHQVRU  FRPSODLQWV E  DIIHFWHG SDWLHQWV NCSs demonstrate normal or mildly slow NCVs, reduced CMAPs, severe disability. DI-CMTC maps to chromosome 1p34-35 due to and reduced or absent SNAPs, and nerve biopsy is consistent with tyrosyl-tRNA synthetase (YARS) gene mutations, and DI-CMTD maps to chromosome 1q22 MPZ gene mutations.23,24 an increasing number of CMT2 subtypes.16 CMT2A is the most common subtype and accounts for 30% X-linked CMT (CMTX) is the second most common form and of CMT2 cases. An early and a late onset have been reported, accounts for approximately 13% of all CMT cases. Males are and some cases are associated with optic atrophy or hearing more severely affected, whereas females may only have a mild loss. The disease is due to mitofusin 2 (MFN2) gene mutations, neuropathy, with no male-to-male inheritance. Clinically it is chromosome 1p36, expressed in and related to mitochondrial similar to CMT1, except for more severe hand weakness and function.17 Approximately 20% of MFN2 mutations are de novo, muscle atrophy. Asymptomatic or mild CNS involvement and accounting for sporadic cases. CMT2B patients have primarily brain magnetic resonance imaging (MRI) white matter changes sensory rather than motor symptoms, including sensory loss and are seen in some cases. CMTX has been mapped to chromosome foot ulcerations, similar to hereditary sensory neuropathy type 1. It is caused by Ras-related protein Rab-7 (RAB7) gene mutations, Connexin 32 (Cx32), an important gap junction radial transport chromosome 3q21. A second CMT2B1 phenotype, mostly SURWHLQ(';¿QGLQJVLQFOXGHGHP HOLQDWLRQDQGD[RQDOIHDWXUHVreported in North Africa, is mapped to chromosome 1q21 due with an intermediate range of NCVs, and, in contrast to CMT1, to Lamin A/C (LMNA) mutations, involved in nuclear envelope nonuniform slow NCVs or dispersion can be seen in some function and allelic to limb-girdle muscular dystrophy.18 patients. Most CMTX cases are X-linked dominant (CMT1X), which is a mixed demyelinating and axonal neuropathy. The CMT2C is rare, with an early age of onset of CMT2 clinical features, associated with skeletal deformities, voice hoarseness, clinical features as CMT1 (except with more hand weakness and DQGUHVSLUDWRU GLI¿FXOW VHFRQGDU WRYRFDOFRUGDQGGLDSKUDJP atrophy), and at times associated with CNS transient symptoms, paresis. The genetic defect involves as of yet unknown gene or abnormal brainstem auditory evoked responses and brain MRI mutation of chromosome 12q23-24.19 CMT2D typically has white matter changes.25,26 CMT2X is a rare X-linked recessive prominent hand muscle weakness and atrophy, variable sensory form, often associated with hearing loss and at times with mental loss, and age of onset in the second or third decade. The disease is mapped to chromosome 7p14 due to Glycyl-tRNA synthetase (GARS) gene mutations which are involved in protein translation. Autosomal Recessive Charcot–Marie–Tooth Disease
CMT2E is a rare subtype which presents in the second or third Autosomal recessive CMT types, also known as CMT4, are less decade with distal lower-limb weakness and muscle atrophy, feet common than the autosomal dominant forms, and accounts for deformities, and can be associated with deafness. It is mapped less than 10% of all CMT cases. They are usually present in higher WR FKURPRVRPH S GXH WR QHXUR¿ODPHQW WULSOHW / 1()/  numbers in certain ethnic groups with high rates of consanguinity. mutations, a protein of axonal transport.20 The disease was initially reported in Tunisian families, and then reported in Turkey and elsewhere. Both demyelinating and axonal The remaining CMT2 subtypes are very rare and reported in CMT4 forms have been recognized by electrophysiological and few families. CMT2F is due to heat-shock protein B1 (HSP27) histological features.28 gene mutation, chromosome 7q11-21, having an adult onset with distal weakness and atrophy of feet and hands. CMT2G maps to The demyelinating CMT4 forms include six subtypes, present chromosome 12q12-13 of unknown gene mutation and has mild with clinical features similar to CMT1, but usually with an severity. CMT2H maps to chromosome 8q21 of unknown gene earlier age of onset and a more severe progressive course, with mutation, CMTI is due to MPZ gene mutation, chromosome almost all cases wheelchair dependent by the third decade. NCVs 1q22, with late age presentation. CMT2J maps to chromosome are variably slow, CMAPs are reduced, and SNAPs reduced 1q22-23 due to another MPZ gene mutation with prominent or unobtainable. Several causative genes mutations have been sensory symptoms, hearing loss, and tonic pupils. CMT2K maps to chromosome 8q13 with ganglioside-induced differentiation of all CMT4 cases. The disease has an early onset and a severe protein 1 (GDAP1) gene mutations. CMT2L maps to chromosome progressive course leading to severe disability, with prominent 12q24 due to HSP8 gene mutations.21,22 foot and spine deformities, and can be associated with vocal cord and diaphragmatic paralysis. The disease has been mapped to Dominant Intermediate Charcot–Marie–Tooth Disease
chromosome 8q13 due to GDAP1 mutations, expressed on both Dominant intermediate CMT (DI-CMT) includes four subtypes neurons and Schwann cell mitochondria, which may explain the characterized by clinical features similar to those of CMT1 and described demyelinating or axonal forms.29 CMT2. EDX studies show intermediate NCVs (median nerve more than 38 m/s) and needle EMG signs of axonal neuropathy. CMT4B1 and CMT4B2 present in infancy, often associated with Nerve biopsy shows histological evidence of both demyelination cranial nerve involvement and a very severe progressive course. and axonal features. DI-CMTA maps to chromosome 10q24-25 Nerve biopsy shows characteristic focally folded myelin. Both due to an unknown gene defect. DI-CMTB maps to chromosome forms are caused by myotubularin-related protein gene (MTMR) 19p12-13 due to dynamin 2 (DNM2) gene mutations, a protein PXWDWLRQV ZKLFK SOD  D UROH LQ QHXUDO PHPEUDQH WUDI¿FNLQJ that plays a key role in receptor–mediate endocytosis, and CMT4B1 has been mapped to chromosome 11q21 caused by presents with severe distal and proximal weakness leading to MTMR2 mutations, and CMT4B2 to chromosome 11p15 caused NEUROMUSCULAR UPDATE II: MIND THE GAP! BETWEEN THEORY AND PRACTICE E  0705 PXWDWLRQV &07& SUHVHQWV LQ WKH ¿UVW GHFDGH mutations. NCVs are markedly slowed due to primary myelin with variable severity and progression, common involvement of SURGXFWLRQ GH¿FLHQF  ZLWK PDUNHG UHGXFWLRQ RI QHUYH ¿EHUV respiratory muscles, and wide range of motor NCV variability minimal myelin, and lack of onion bulb formation.
in the same limb. The disease has been mapped to chromosome 5q23-33 due to KIAA1985 protein gene mutations.30 Hereditary Neuropathy with Liability to Pressure Palsy
HNPP is an autosomal dominant hereditary neuropathy and &07' ZDV ¿UVW UHSRUWHG RQO  LQ J SV  SRSXODWLRQV LQ /RP usually presents in the second or third decade. It may have city Bulgaria, thus named HMSN-Lom, then in other European variable neuropathic manifestations; however, the typical countries including Spain, Italy, and Slovenia. Clinically, it is presentation is recurrent painless focal neuropathies at the usual similar to other CMT4 forms, with deafness and tongue atrophy. entrapment sites, usually after minor compression or trauma, or Motor NCVs are markedly slowed and histological features include onion bulbs, axonal loss, and inclusions. The disease has neuropathies such as in the facial nerve, or recurrent painless been mapped to chromosome 8q24 due to N-myc downstream- brachial plexopathy are described. Some patients have mild regulated gene 1 (NDRG1) mutation, a Schwann cell signaling sensorimotor polyneuropathy, feet deformities and absent ankle protein.31 Other forms of demyelinating CMT4 are rare and include CMT4E due to early growth response 2 (EGR2) mutations, which Usually the recurrent mononeuropathies recover in weeks; also cause CMT1D, and CMT4F caused by mutations of the however, repeated episodes may leave persistent neurologic periaxin gene (PRX) with extremely slow NCVs.
GH¿FLW 1&6V RIWHQ UHYHDO PLOGO  VORZ 1&9V LQFUHDVHG GLVWDOlatencies, and conduction block across compression sites. The The axonal CMT4 forms are rare, and only two causative genes hallmark of the nerve biopsy is focal sausage-like thickening of KDYH EHHQ LGHQWL¿HG7KH ¿UVW /DPLQ$& /01$  PXWDWLRQV the myelin sheath (named tomacula) and variably reduced large causes a variety of other inherited neuropathy (CMT2B1) and P HOLQDWHG QHUYH ¿EHUV DQG D[RQDO ORVV RI PLOG VHYHULW  7KH muscular dystrophy (i.e., Emery–Dreifuss and limb-girdle disease has been mapped to chromosome 17p11.2-12 due to 1.5 Mb muscular dystrophy). The disease presents with severe distal deletion in the PMP22 gene in 85% of cases, causing a loss of weakness and muscle atrophy of lower more than upper limbs function of the PMP22, an inverse to PMP22 gene duplication seen in CMT1A with a toxic gain of function. Sporadic HNPP cases are due to spontaneous de novo deletion resulting from paternal mutations, which more often causes demyelinating CMT4 origin unequal crossing during meiosis.36-38 Rare pedigrees with subtypes.32 Giant axonal neuropathy is a recessive disorder of HNPP did not map to chromosome.17 the PNS and CNS, caused by mutations in the gigaxonin gene (GAN1), chromosome 16q24. The disease presents in infancy with Hereditary Neuralgic Amyotrophy
Hereditary neuralgic amyotrophy (HNA) is a rare autosomal dominant inherited recurrent brachial plexopathy which often nerve biopsy shows characteristic massive axonal enlargement starts in childhood. It presents with recurrent episodes of pain IROORZHG E  ZHDNQHVV DQG VHQVRU  GH¿FLW LQ WKH DIIHFWHG XSSHUlimb, resembling sporadic neuralgic amyotrophy. Most patients Dejerine-Sottas Syndrome (HMSN-III or CMT3)
recover over weeks to few months; however, attacks often leave DSS, also known as HMSN-III or CMT3, is a severe hypertrophic demyelinating neuropathy with infancy or early childhood onset. dysmorphic features. The EDX examination shows mostly axonal The disease presents with distal weakness, hypotonia, delayed features, but demyelinating features have been described as well. The disease has been mapped to chromosome 17q25 due is progressive and extends to the upper limbs and proximal to mutations in Septin 9 gene (SEPT9), which is involved in muscles which diminishes ambulation ability and most patients formation of neuronal cytoskeleton and cell division.39 DUH ZKHHOFKDLU GHSHQGHQW /DUJH ¿EHU VHQVRU  ORVV UHVXOWV LQataxia, and sensorineural hearing loss is common. Peripheral Rational Approach for Diagnosis and
nerves are enlarged, determined by palpation or by imaging of the brachial plexus and lumbar spine, and elevated cerebrospinal ÀXLG SURWHLQ PD  EH SUHVHQW 1HUYH ELRSV  VKRZV ODUJH RQLRQ The evaluation of patients with suspected hereditary neuropathy bulb formation, hypomyelination and marked loss of myelinated includes a detailed neurological history, followed by an informed EDX study, and then a rationally selected genetic testing. Known reduced CMAP amplitude and variable degree of denervation. causes of acquired peripheral neuropathy should be excluded as Genetic studies showed various point mutations in MPZ, PMP22, indicated. Obtaining a careful clinical and family history, going or EGR2 genes, mapped to chromosomes 1q22-23, 17p11 and back at least three generations when possible is critical, not only 10q21-22, respectively. Most are heterozygous suggesting to support diagnosis of inherited neuropathy, but also to determine autosomal dominant inheritance. Many cases are sporadic due to who is at risk for developing neuropathy. A strongly positive de novo spontaneous mutations. Recessive cases of DSS are less family history including both genders supports dominantly common and have been linked to PRX, EGR2, and GDAP1.34,35 inherited neuropathy, and male-to-male transmission excludes X-linked inheritance; however, family history is often negative Congenital hypomyelinating neuropathy (CHN) likely represents in autosomal recessive disorders. Likewise, while a positive the most severe DSS spectrum and is mostly linked to MPZ family history is invaluable, caution must be taken in a negative family history since sporadic de novo mutations can start in any Table 2. Stepwise rational approach for diagnosis of inherited neuropathy
particular patient. NCSs are indispensable tool in characterizing and differentiating demyelinating from axonal neuropathy, with Detailed clinical history and possible inheritance mode.
Informed electrodiagnostic examinations, including at-risk motor and sensory potential amplitudes and normal or subnormal family members, if warranted.
NCVs. Additionally, most patients with inherited demyelinating Select genetic testing strategies (demyelinating, X-linked, neuropathy have diffuse, uniform, and symmetrical slowed axonal, autosomal recessive, or intermediate Charcot– NCVs, without block or temporal dispersion, and HNPP has Marie–Tooth disease phenotype panels).
a unique pattern of NCS abnormalities. In contrast, acquired demyelinating neuropathies such as CIDP show asymmetric slowing of NCVs, conduction block, and temporal dispersion. However, NCVs in DI-CMT forms are in the intermediate UDQJHDWWLPHVGLI¿FXOWWRGLVWLQJXLVKIURP&07)XUWKHUPRUH 1. Holmberg BH. Charcot-Marie-Tooth disease in northern Sweden: an nonuniform slowing of NCVs, conduction block, and dispersion epidemiological and clinical study. Acta Neurol Scand 1993;87:416- can be seen in some patients with dominant CMTX cases and a few with MPZ and EGR2 mutations, causing confusion with the 2. King S, Lupski JR. Charcot-Marie Tooth disease. Eur J Hum Genet acquired demyelinating neuropathies.40 3. Reilly MM. Sorting out the inherited neuropathies. Pract Neurol Genetic testing, despite its complexity, has become an indispensable tool in the diagnosis of hereditary neuropathy. 4. Skre H. Genetic and clinical aspects of Charcot-Marie-Tooth's disease. Clin Genet 1974;6:98-118.
end to the patient and their families' quest and permits genetic 5. Gilliatt RW, Thomas PK. Extreme slowing of nerve conduction in counseling and an informed discussion of the prognosis, potential peroneal muscular atrophy. AA Phys Med 1957;4:104-106.
complications, and management even if there is no curative 6. Dyck PJ, Lambert EH. Lower motor and primary sensory neuron treatment, keeping in mind that a negative genetic test does not diseases with peroneal muscular atrophy; neurologic, genetic, and exclude a genetic disease. Genetic tests are quite expensive, and (OHFWURSK VLRORJLF ¿QGLQJV LQ KHUHGLWDU  SRO QHXURSDWKLHV $UFK they are available commercially as individual tests, with partial and complete panels based on EDX features or inheritance mode. 7. Hattori N, Yamamoto M, Yoshihara T et al. Demyelinating and axonal A rational approach guided by the clinical phenotype, inheritance features of Charcot-Marie-Tooth disease with mutations of myelin- pattern, and EDX features in selecting the appropriate test is related proteins (PMP22, MPZ, and Cx32): a clinicopathological recommended to reduce the cost and improve the diagnostic yield study of 205 Japanese patients. Brain 2003;126:134-151.
and should focus on the most common abnormalities. Most cases 8. Dyck PJ, Chance P, Lebo R, et al. Hereditary motor and sensory of CMT are autosomal dominant, with CMT1 twice as common, QHXURSDWKLHV ,Q ' FN 3$ 7KRPDV 3. *ULI¿Q -: /RZ 3$ the majority of which are CMT1A (70%) followed by CMT1B Poduslo JF, eds. Peripheral neuropathy, Vol 2, 3rd ed. Philadelphia: (20%). CMT2 is the second most common, of which CMT2A WB Saunders; 1993. pp 1094-1136.
phenotype represents 30% of cases. X-linked CMT represents 9. Vance JM, Nicholson GA, Yamaoka LH et al. Linkage of Charcot- 13% of CMT cases, and a search for Cx32 (GJB1) mutations is Marie-Tooth neuropathy type 1a to chromosome 17. Exp Neurol are less affected). Several population-based studies have shown 10. Lupski JR, deOca-Luna RM,Slaugenhaupt S, et al. DNA duplication that PMP22 duplication, PMP22 deletion and Cx32 (GJB1), associated with Charcot-Marie-Tooth disease type 1 A. Cell MPZ, and MFN2 mutations account for 65-70% of CMT cases. The respective frequencies are PMP22 duplication: 52-78%, 11. Roa BB, Garcia GA Suter U, et al. Evidence for Charcot-Marie- Cx32: 6-21%, MPZ: 3-11%, PMP point mutations: 2-3%, NEFL: Tooth disease type 1A association with point mutation in the PMP22 1-5%, and MFN2: 33% of CMT2 cases. The rest are much less gene. N Engl J Med 1993;5:189-194.
frequent (each <1%).41,42 12. Hayasaka K, Himoro M, Sato W, et al. Charcot-Marie-Tooth neuropathy type 1B is associated with mutations of the myelin PO gene. Nat Genet 1993;5:31-34.
13. Plante-Bordeneuve V, Guiochon-Mantel A, Lacroix C, et al. The IPNs are among the most prevalent inherited neurologic Roussy-Levy family: from the original description to the gene. Ann disorders, and, with current advances in molecular biology and genetic testing, the clinical spectrum of phenotype/genotype has 14. Boerkoel CF, Takashima H, Garcia C, et al. Charcot-Marie- been expanding enormously. Genetic testing for several subtypes Tooth disease and related neuropathies: mutation distribution and is now commercially available, although the genetic defects of genotype-phenotype correlation. Ann Neurol 2002;51(2):190-201.
many other remain unknown. A stepwise rational approach, VKRZQ LQ 7DEOH  IDFLOLWDWHV UHDFKLQJ D VSHFL¿F GLDJQRVLV DQG light (NEFL) mutation Glu397Lys is associated with a clinically reduces the cost.
and morphologically heterogeneous type of Charcot-Marie-Tooth neuropathy. Neuromuscul Disord 2004;14(2):147-157.
16. Bienfait HMF, Verhamme C, van Schaik IN, et al. Comparison of CMT1A and CMT2: similarities and differences. J Neurol 2006;253(12):1572-1580.
NEUROMUSCULAR UPDATE II: MIND THE GAP! BETWEEN THEORY AND PRACTICE 17. Zuchner S, Mersiyanova IV, Muglia M, et al. Mutations in the 31. Hunter M, Bernard R, Freitas E, et al. Mutation screening of the mitochondrial GTPase mitofusin 2 cause Charcot-Marie-Tooth N-myc downstream-regulated gene 1 (NDRG1) in patients with neuropathy type 2A. Nat Genet 2004;36:449-451.
Charcot-Marie-Tooth disease. Hum Mutat 2003;22:129-135.
18. Chaouch M, Allal Y, De Sandre-Giovannoli A, et al. The phenotype 32. Chaouch M, Allal Y, De Sandre-Giovannoli A, Vallat JM, et al. The manifestations of autosomal recessive axonal Charcot-Marie- phenotypic manifestations of autosomal recessive axona Charcot- Tooth due to a mutation in lamin A/C gene. Neuromuscul Dis Marie-Tooth due to mutation in lamin A/C gene. Neuromuscul 19. Dyck PJ, Litchey WJ, minnerath S, et al. Hereditary motor and sensory neuropathy with diaphragam and vocal cord paresis. Ann in patients with giant axonal neuropathy. Neurology 2004;62:13-16.
34. Plante-Bordeneuve V, Said G. Dejerine-Sottas disease of infancy. 20. Irobi J, de Jonghe P, Timmerman V. Molecular genetics of distal Muscle Nerve 2002;26:608-621.
hereditary motor neuropathies. Hum Mol Genet 2004;13:R195-202.
35. Boerkoel CF, Takashima H, Stankiewicz P, et al. Periaxin mutations 21. Nicholson GA. The dominantly inherited motor and sensory cause recessive Dejerine-Sottas neuropathy. Am J Hum Gen neuropathies: clinical and molecular advances. Muscle Nerve 36. Amato AA, Gronseth G, Callerence K, et al. Tomaculus neuropathy: 22. Szigeti K, Lupski JR. Charcot-Marie-Tooth disease. Euro J Hum A clinical and electrophysiological study in patients with and without 1.5 Mb deletion to chromosome 17p11.2. Muscle Nerve 23. Zuchner S, Noureddine M, Kennerson M, et al. Mutations in the plecstrin homology domain 2 causes dominant intermediate 37. Verhagen WIM, Gabreels-Festen AAWM, van Wensen PJM, et Charcot-Marie-Tooth disease. Nat Genet 2005;37(3):289-294.
al. Hereditary neuropathy with liability to pressure palsies. A 24. Jordanova A, Thomas FP, Guergueltcheva V, et al. Dominant clinical, electrophysiological, and morphology study. J Neurol Sci intermediate Charcot-Marie-Tooth type C maps to chromosome 1p34-p35. Am J Hum Genet 2003;73(6):1423-1430.
38. Li J, Krajewski K, Lewis RA, et al. Loss of function phenotype 25. Hahn AF, Brown WF, Koopman WJ, et al. X-linked dominant of hereditary neuropathy with liability to pressure palsies. Muscle hereditary motor and sensory neuropathy. Brain 1990;113:1511- 39. Kuhlenbaumer G, Hannibal MC, Nelis E, et al. Mutations in SEPT9 26. Paulson HL, Garbern JY, Hoban TF, et al. Transient central nervous cause hereditary neuralgic amyotrophy. Nat Genet 2005;37:1044- system white matter abnormality in X-linked Charcot-Marie-Tooth disease. Ann Neurol 2002;52:429-434.
40. Cutierrez A, England J, Sumner AJ, et al. Unusual 27. Ionasescu VV, Trafatter J, Haines JL, et al. X-linked recessive Charcot-Marie-Tooth neuropathy-clinical and genetic study. Muscle Tooth disease. Muscle Nerve 2000;23:182-188.
41. Boerkoel CF, Takashima H, Garcia CA, et al. Charcot-Marie- 28. Duboourg O, Azzedine H, Verny C, et al. Autosomal-recessive forms Tooth disease and related neuropathies mutation distribution and of demyelinating Charcot-Marie-Tooth disease. Neuromolecular genotype-phenotype correlation. Ann Neurol 2002;51:190-201.
Med 2006;8:75-86.
42. Bort S, Nelis E, Timmerman V, et al. Mutational analysis of the 29. Nicholson G, Ouvrier R. GDAP1 mutations in CMT4: axonal MPZ, PMP22 and Cx32 genes in patients of Spanish ancestry and demyelinating phenotypes. "The exception proves the rule." with Charcot-Marie-Tooth disease and hereditary neuropathy with liability to pressure palsies. Hum Genet 1997;99:746-754.
30. Ouvrier R, Geevasingha N, Ryan MM. Autosomal-recessive and X-linked forms of hereditary motor sensory neuropathy in childhood. Muscle Nerve 2007;36:131-143.
NEUROMUSCULAR UPDATE II: MIND THE GAP! BETWEEN THEORY AND PRACTICE Genetic Testing for Limb-Girdle and
Anthony A. Amato, MD
Director, Neuromuscular Division and Clinical Neurophysiology Laboratory Director, Partners Neuromuscular Medicine Fellowship Program INTRODUCTION: CASE STUDY
A 43-year-old right-handed woman with a history of cardiac OHIW ZKHUH DSSURSULDWH  QHFN ÀH[RUV  QHFN H[WHQVRUV  arrhythmias gradually developed bilateral foot drop about 2.5 VKRXOGHU DEGXFWRUVHOERZ ÀH[RUVHOERZ H[WHQVRUV  ZULVW stairs. She has also noted that her hands are clumsy and has had  DEGXFWRU GLJLWL PLQLPL  KLS ÀH[RUVDEGXFWRUVH[WHQVRUV both solids and liquids for a few years. She has not lost any weight  NQHH H[WHQVRUV  NQHH ÀH[RUV  DQNOH GRUVLÀH[RUV and reports she has always been very thin. She has not had any shortness of breath or dysarthria. There is no neck/back/radicular pain, cramps, or muscle stiffness. She has no history of blurred Light touch, pinprick, and proprioception were intact. There was vision or cataracts.
normal vibratory perception in the toes and ankles bilaterally. 5RPEHUJ¶V WHVW ZDV QHJDWLYH 'HHS WHQGRQ UHÀH[HV ZHUH  DQG Her past medical history is remarkable for a history of cardiac symmetric at the biceps, triceps, brachioradialis, and knees; they were absent at the ankles. There was no Hoffman's sign bilaterally. required placement of a pacemaker and an automatic implantable 3ODQWDU UHVSRQVHV ZHUH ÀH[RU ELODWHUDOO  &RPSOH[ PRWRU VNLOOV FDUGLRYHUWHU GH¿EULOODWRU $,&'  D IHZ HDUV DJR 6KH UHSRUWV revealed normal coordination. She had a steppage gait. She was having symptoms of palpitations and lightheadedness since she was a teenager. She had an episode of cardiac arrest, then had walking in tandem. She was able to rise from a chair without the use of the hands.
and was on Coumadin [warfarin] for some time.) There is no other family history of cardiopathy or neuromuscular disease of which Nerve Conduction Studies and Needle
she is aware.
Electromyography (EMG) Revealed:
On examination, there was no kyphosis, scoliosis, pes cavus, ‡ Increased insertional and spontaneous activity in the following or hammertoes. Cranial nerves II-XII were intact. Motor muscles: increased insertional activity, positive sharp waves, examination revealed normal muscle tone. There was no evidence of fasciculations. Muscle atrophy was noted in the hand intrinsics tibialis anterior, and EDC; increased insertional activity, bilaterally. There was no scapular winging. There was no action or percussion myotonia or paramyotonia. Manual muscle testing thoracic paraspinals; and increased insertional activity in the revealed the following Medical Research Council scores (right/ right vastus medialis.
‡ Polyphasic motor unit action potentials (MUAPs) of brief With so many different types of muscular dystrophies and the duration and small amplitude with early recruitment were seen in the right FDI and tibialis anterior. MUAPs were of of dystrophy even within individual families, the evaluation of normal morphology and recruitment in the other sampled patients presenting with weakness can be quite daunting. However, rather than ordering every genetic test possible or performing a muscle biopsy initially on every patient, an approach to ordering A muscle biopsy of the EDC that was consistent with a tests based on clinical phenotype, pattern of muscle involvement, P R¿EULOODU P RSDWK  ZDV SHUIRUPHG *HQHWLF WHVWLQJ IRU inheritance pattern, age of onset, and associated manifestions P R¿EULOODUP RSDWK ZDVVXEVHTXHQWO RUGHUHGDQGDPXWDWLRQ (i.e., early contractures, cardiac or respiratory involvement) can LQ WKH '(6 JHQH WKDW HQFRGHV IRU GHVPLQ ZDV LGHQWL¿HG very useful.
(Heterozygous c.1360C>T [p.Arg454Trp], Exon 8, DES). This JHQHKDVEHHQSUHYLRXVO DVVRFLDWHGZLWKP R¿EULOODUP RSDWK  The Guideline committee sponsored by the American Association and cardiomyopathy.
of Neuromuscular & Electrodiagnostic Medicine and American Academy of Neurology has been working the past several years in As this is an autosomal dominant disorder and can be associated putting together a practice parameter, including an algorithm for with sudden cardiac death from arrhythmia, her children testing for LGMDs. Hopefully, this guideline should be published underwent genetic testing. Her younger daughter (13 years old) soon, but below is a summary of what the committee will be was found to have a mutation in the desmin gene. Her EKG and recommending based on their extensive review of the literature.
echocardiogram were also abnormal. The daughter underwent pacemaker and AICD placement.
The most important step in this process is recognizing the pattern of muscle involvement: which muscle groups are weak, MUSCULAR DYSTROPHIES: OVERVIEW
is there atrophy, is there hypertrophy, is there scapular winging? It is useful to classify the patterns of weakness into limb-girdle, The muscular dystrophies are a group of hereditary, progressive humeroperoneal, oculopharyngeal, and distal. This review does not address dystrophies with oculopharyngeal weakness. Next to and replacement by connective and fatty tissue. The different consider is the inheritance pattern: is this an autosomal recessive, forms of muscular dystrophy result from mutations affecting dominant, or X-linked disorder (remembering that each of these proteins localizable to the sarcolemma, nucleus, basement can occur in sporadic patients)? Subsequently, which other PHPEUDQH DQG H[WUDFHOOXODU PDWUL[ VXUURXQGLQJ PXVFOH ¿EHUV features can help distinguish subtypes: cardiac or respiratory the sarcomere, and nonstructural enzymatic proteins. The clinical involvement, creatine kinase levels, needle EMG abnormalities onset of the dystrophy may be evident at birth as in congenital such as myotonic discharges, or muscle biopsy features? With muscular dystrophies or not develop until late adulthood. this in mind, the following is how patients with these dystrophies Historically, the dystrophies were believed to be different from should be approached.
other forms of myopathy (e.g., congenital muscular dystrophies DQGP RSDWKLHVGLVWDOP RSDWKLHVP R¿EULOODUP RSDWKLHVDQG LIMB-GIRDLE PATTERN OF WEAKNESS
hereditary inclusion body myopathies). However, with advances in molecular genetics the distinction between what constitutes a If There Is an Autosomal Dominant Inheritance
muscular dystrophy from other hereditary myopathies has become ‡ ,Q SDWLHQWV ZLWK VXSHULPSRVHG DQNOH GRUVLÀH[RU ZHDNQHVV (foot drop), cardiomyopathy, respiratory involvement, needle inheritance, and pattern of weakness (Table). For example, those EMG with "pseudomyotonic" discharges, or previous muscle that present at birth are called congenital muscular dystrophies. ELRSV  ZLWK IHDWXUHV RI P R¿EULOODU P RSDWK  0)0  Dystrophies also have been named based on the patterns of muscle consider genetic testing for mutations in the genes for myotilin involvement including limb-girdle muscular dystrophy (LGMD), (LGMD1A), DNAJB6 (LGMD1D), desmin (LGMD1E), facioscapulohumeral dystrophy, oculopharyngeal muscular dystrophy, distal myopathy/dystrophies, and humeroperoneal dystrophy. Within the distal muscular dystrophies, ‡ In patients manifesting rippling muscles, check for mutations VXEFODVVL¿FDWLRQV KDYH EHHQ EDVHG RQ LQKHULWDQFH SDWWHUQ DJH in the gene for caveolin-3 (LGMD1C).
RI RQVHW DQG WKH VSHFL¿F PXVFOH JURXSV LQLWLDOO  DIIHFWHG HJthe Markesbery-Griggs, Udd, and Laing types of distal myopathy ‡ In patients with early involvement of humeroperoneal have preferential involvement of the anterior tibial muscles, muscles, contractures, and cardiomyopathy, check for Miyoshi myopathy the gastrocnemius, and Welander myopathy mutations in the gene for lamin A/C (LGMD1B).
the extensor forearm muscles). Dystrophies associated with proximal greater than distal weakness are called LGMDs. The ‡ In patients with proximal and/or distal weakness, myotonic LGMDs inherited in an autosomal dominant fashion are termed discharges on needle EMG, past medical or family history LGMD type 1 (LGMD1), while autosomal recessive dystrophies of Paget disease, dementia, or motor neuron disease, check for mutations in the gene for valosin containing protein based on genotype differences (e.g., LGMD1A, LGMD1B, etc.).
(hereditary inclusion body myopathy with Paget's disease and frontotemporal dementia, or hIBMPFD).
NEUROMUSCULAR UPDATE II: MIND THE GAP! BETWEEN THEORY AND PRACTICE X-linked dystrophies
Duchenne/Becker muscular
0 R¿EULOODUP RSDWKLHV
Hereditary IBM
AR-Hereditary IBM
H-IBM with FTD and Paget's Continued on next page Early onset distal Myopathy with Kelch-line 9
(Drosophila) mutation
Other dystrophies
EDMD3
Muscular dystrophy with $' DXWRVRPDO GRPLQDQW $5 DXWRVRPDO UHFHVVLYH %$* %&/DVVRFLDWHG DWKDQRJHQH  '1$-% '1$- KRPRORJ VXEIDPLO  % PHPEHU ('0' (PHU ±'UHLIXVV PXVFXODU G VWURSK  )+/ )RXUDQGDKDOI /,0 SURWHLQ  ).53 )XNXWLQUHODWHG SURWHLQ )7' URQWRWHPSRUDO GHPHQWLD*1( 8'31DFHW OJOXFRVDPLQH HSLPHUDVHQDFHW OPDQQRVDPLQH NLQDVH +,%0 KHUHGLWDU  LQFOXVLRQ ERG  P RSDWK  /*0' OLPEJLUGOH PXVFXODUG VWURSK /80$ 7UDQVPHPEUDQHSURWHLQ0 +& P RVLQKHDY FKDLQ320*Q7 2PDQQRVHȕ1DFHW OJOXFRVDPLQ OWUDQVIHUDVH3207 2PDQQRV OWUDQVIHUDVH3207 2PDQQRV OWUDQVIHUDVH7,$ 7FHOOUHVWULFWHGQXFOHDUDQWLJHQ75,0 (XELTXLWLQOLJDVH9&3 YDORVLQFRQWDLQLQJSURWHLQ;5 ;OLQNHGUHFHVVLYH=$63 =EDQGDOWHUQDWLYHO VSOLFHG3'=PRWLIFRQWDLQLQJSURWHLQ0RGL¿HGZLWKSHUPLVVLRQIURP$PDWR$$5XVVHOO-1HXURPXVFXODUGLVHDVH1HZ<RUN0F*UDZ+LOOS7DEOH ‡ In patients with distal weakness, early cataracts, clinical ‡ In patients with northern European ancestry, calf hypertrophy, myotonia, cardiac arrhythmia, sleep apnea symptoms, scapular winging, and early cardiac and pulmonary cognitive issues, and myotonic discharges on needle EMG, involvement, consider genetic testing for FKRP mutations consider genetic testing for myotonic dystrophy type 1.
‡ In patients with proximal weakness, myalgias, early cataracts, ‡ In patients with history of epidermolysis bullosa (blistering clinical myotonia, and cardiac arrhythmia myotonic/ skin) or pyloric atresia, consider genetic testing for mutations pseudomyotonic discharges on needle EMG, consider genetic testing for myotonic dystrophy type 2.
‡ If genetic testing is normal, perform muscle biopsy and ‡ If there is no characteristic clinical phenotype present, then immunostaining for dystrophin, sarcoglycans, merosin, perform a muscle biopsy.
ĮG VWURJO FDQ G VIHUOLQ DQG FDYHROLQ &RQVLGHUimmunoblot for dystrophin, calpain-3, and dysferlin, if the ‡ If the biopsy shows features MFM, consider checking for above are negative.
mutations seen with MFM as mentioned above.
‡ If immunostaining/immunoblots are abnormal, consider ‡ If the biopsy just shows rimmed vacuoles and inclusions, consider hIBMPFD.
‡ ,I WKH ELRSV  UHYHDOV RQO  QRQVSHFL¿F IHDWXUHV FRQVLGHU ‡ ,I WKH ELRSV  VKRZV PDQ  PXVFOH ¿EHUV ZLWK QXPHURXV testing for late-onset Pompe disease (e.g., dried blood spot) or genome-wide screening.
nuclear clumping, consider testing for myotonic dystrophy type 2.
If There Is an X-Linked Inheritance Pattern or
‡ If biopsy is nondiagnostic, consider genome-wide screening.
a Sporadic Male
‡ Consider genetic testing for mutations in the dystrophin gene If There Is an Autosomal Recessive Inheritance
(Duchenne or Becker muscular dystrophy). If this is negative, perform a muscle biopsy, immunostaining, immunoblots, and subsequent testing, as above.
‡ In patients from Great Britain, southern or eastern Europe, and Brazil, with scapular winging, no calf hypertrophy (may ‡ If there are features of MFM or reducing bodies on the muscle have atrophy), and no heart or ventilatory weakness, consider biopsy, consider genetic testing for FHL1 mutations.
genetic testing for calpain-3 mutations (LGMD2A).
‡ If the biopsy shows non-rimmed vacuoles, consider Danon ‡ In patients with atrophy of calf muscles or an inability to stand disease and X-linked myopathy with excessive autophagia.
on tip toes, consider genetic testing for ANO5 (LGMD2L) or dysferlin (LGMD2B) mutations.
NEUROMUSCULAR UPDATE II: MIND THE GAP! BETWEEN THEORY AND PRACTICE ‡ If the biopsy is nondiagnostic and genetic testing is ‡ If biopsy just shows rimmed vacuoles and inclusions, unrevealing, consider genome-wide screening.
consider hIBMPFD.
‡ In women with a possible X-linked disorder (familial ‡ In patients with the above features presenting in childhood presentation with males more affected than females or or early adulthood (<40 years), consider genetic testing for sporadic presentation), check for a mosaic appearance of dystrophin by immunohistochemistry, if there are no affected males available for genetic testing for dystrophin.
‡ In patients with early adult onset of distal weakness with foot drop, neck weakness, cardiomyopathy but no vacuoles HUMEROPERONEAL PATTERN OF
on biopsy, consider genetic testing for mutations in MyHC7 WEAKNESS (EMERY–DREIFUSS MUSCULAR
(Laing myopathy).
‡ In patients with childhood onset of distal weakness with foot drop, neck weakness, cardiomyopathy but no vacuoles on ‡ In patients with a humeroperoneal pattern of weakness, early biopsy, consider checking for mutations in the gene for KLHL9.
elbow/ankle contractures, rigid spine, or cardiac involvement and an autosomal dominant inheritance pattern, consider ‡ ,IELRSV UHYHDOVRQO QRQVSHFL¿FIHDWXUHVRUJHQHWLFWHVWLQJ genetic testing for mutations involving lamin A/C. In patients is unrevealing consider genome-wide screening.
with the above presentation but an X-linked inheritance pattern, consider genetic testing for mutations in emerin.
If There Is an Autosomal Recessive Inheritance
‡ If genetic testing is negative, perform muscle biopsy to look for MFM features, and, if present, consider genetic testing for FHL1 mutations (particularly if X-linked and there ‡ In patients with early adult onset calf weakness/atrophy, are reducing bodies); if this is negative, check for desmin ZLWK QR YDFXROHV RU IHDWXUHV RI P R¿EULOODU P RSDWK  RQ biopsy, consider genetic testing for mutations in the genes for dysferlin and ANO5 (Miyoshi myopathy).
‡ In patients with the above clinical features and also laxity of distal joints and protuberant calcanei but no cardiac ‡ In patients with early adult onset foot drop and rimmed involvement, consider genetic testing for collagen VI vacuoles on muscle biopsy, consider genetic testing for mutations (i.e., Ullrich congenital muscular dystrophy mutations in the GNE gene (hereditary inclusion body if congenital onset and autosomal recessive inheritance, Bethlem myopathy if later onset and autosomal dominant inheritance).
‡ In patients with early adult onset foot drop and nemaline rods on muscle biopsy, consider genetic testing for mutations in ‡ ,IELRSV UHYHDOVRQO QRQVSHFL¿FIHDWXUHVRUJHQHWLFWHVWLQJ the nebulin gene.
is unrevealing consider genome-wide screening.
‡ ,IELRSV UHYHDOVRQO QRQVSHFL¿FIHDWXUHVRUJHQHWLFWHVWLQJ is unrevealing consider genome-wide screening.
'LVWDOP RSDWKLHVPD EHFODVVL¿HGLQWRHDUO   HDUVRIDJH and late (>40 years) onset.
If There Is an X-Linked Inheritance Pattern
If There Is an Autosomal Dominant Inheritance
‡ In patients with distal weakness with foot drop, with or without cardiomyopathy, particularly if there are reducing bodies on muscle biopsy, consider genetic testing for mutations ‡ Perform a muscle biopsy to look for rimmed vacuoles, FHL1 gene (reducing body myopathy). If biopsy reveals other features of MFM, reducing bodies, and abnormal RQO  QRQVSHFL¿F IHDWXUHV RU JHQHWLF WHVWLQJ LV XQUHYHDOLQJ accumulation of lipid or glycogen.
consider genome-wide screening.
‡ In patients with mid-to-late adult onset (>40 years) of Again, it is important to emphasize that autosomal dominant, distal weakness with early foot drop, with or without autosomal recessive, or X-linked myopathies may present as cardiomyopathy, ventilatory muscle weakness, needle sporadic cases.
EMG showing "pseudomyotonic" discharges, and biopsy consistent with MFM, consider genetic testing for myotilin /*0'$  =$63 GHVPLQ /*0'(  Į% FU VWDOOLQ¿ODPLQ&'1$-% /*0'' DQGWLWLQ These recommendations are based on the following Guideline that is in press: Evidence-based Guideline: Diagnosis and Treatment of Limb-Girdle, Humeroperoneal, and Distal Muscular Dystrophies Report of the PIRP of the American Association of Neuromuscular & Electrodiagnostic Medicine and the Guideline Development 6XEFRPPLWWHHRIWKH$PHULFDQ$FDGHP RI1HXURORJ  (1) Pushpa Narayanaswami, MBBS, DM, FAAN; (2) Michael Weiss, MD, FAAN, MD; (3) Duygu Selcen, MD, PhD; (4) William David, MD, PhD; (5) Elizabeth Raynor, MD; (6) Gregory Carter, MD; (7) Matthew Wicklund, MD, FAAN; (8) Richard J. Barohn, MD, FAAN; (9) Erik Ensrud, MD; (10) Robert C. Griggs, MD, FAAN; (11) Gary Gronseth, MD, FAAN; (12) Anthony A. Amato, MD, FAAN.
NEUROMUSCULAR UPDATE II: MIND THE GAP! BETWEEN THEORY AND PRACTICE Amanda C. Peltier, MD
Assistant Professor, Vanderbilt University Medical Center Gerald J. Herbison, MD
Professor, Department of Rehabilitation Medicine Jefferson Medical College, Thomas Jefferson University Björn E. Oskarsson, MD
Assistant Professor of Clinical Neurology Director, Multidisciplinary ALS Clinic University of California Davis Medical Center VIGNETTE ONE
A 59-year-old woman with a history of type 1 diabetes for 25 ordered (e.g., deep breathing, Valsalva maneuver with years presents with complaints of dizziness when standing, heat continuous blood pressure, and sudomotor testing)? intolerance, and neuropathy pain. She has a history of nephropathy, A. Normal, this is not an autonomic neuropathy.
retinopathy, and Charcot foot. She has fatigue during the day and B. Abnormal, heart rate response to deep breathing only palpitations when active. The following results are obtained in will be absent or reduced.
C. Abnormal, heart rate response to deep breathing will be absent or reduced, Valsalva maneuver will show lack of Length of time Heart rate (bpm) Blood pressure (mmHG)
heart rate elevation and loss of phase II late and phase IV blood pressure components.
D. Abnormal, heart rate response to deep breathing will be absent or reduced, Valsalva maneuver will show lack of heart rate elevation and loss of phase II late and phase IV blood pressure components, and sudomotor testing will show distal loss of sweat in length dependent gradient.
1B. Which autonomic test results show the earliest changes seen in patients with diabetic autonomic neuropathy?A. Impaired heart rate response to deep breathing.
B. Impaired blood pressure response to valsalva maneuver.
C. Heart rate elevation with standing.
D. Orthostatic blood pressure decrease with submaximal heart rate elevation with standing.
NEUROMUSCULAR VIGNETTES 1C. Which intervention is MOST appropriate? VIGNETTE TWO
A. Behavioral changes such as toe raise, leg cross, step up A 50-year-old man presents with pain in his feet which gets worse to increase blood pressure when standing, raising head with activity and at night when lying in bed. He has a history of elevated cholesterol for which he takes a statin and was found B. Trial of medications such as midodrine 5-10 mg three on laboratory workup to have impaired glucose tolerance, with a times a day as needed if no supine hypertension present.
C. Counseling patient to drink two 8 oz glass of water in the morning before rising to raise systolic blood pressure pin to his calves bilaterally. A needle electromyography (EMG) examination shows the following: D. All of the above.
Motor Nerve Conduction Studies
Nerve and site
Latency (ms)
Amplitude (mV) Segment
Peroneal nerve (right)Ankle Tibial nerve (right)Ankle Ankle-popliteal fossa Tibial nerve (left)Ankle latency (ms)
latency (ms)
Tibial nerve (right) Sensory Nerve Conduction Studies
Nerve and site
Latency (ms)
Sural nerve (right)Lower leg 2A. What is the MOST sensitive test used to diagnose this patient's disorder?A. 4XDQWLWDWLYHVXGRPRWRUD[RQUHÀH[WHVW 46$57 B. Quantitative sensory testing.
C. Distal and proximal leg skin biopsies with intraepidermal D. Sympathetic skin response.
A QSART was performed which shows the following (right): NEUROMUSCULAR UPDATE II: MIND THE GAP! BETWEEN THEORY AND PRACTICE 2B. What other laboratory tests should be considered to discover possible etiologies?A. Vitamin B12 level.
B. Serum protein and urine protein electrophoresis.
C. Transthyretin genetic testing.
D. All of the above.
2C. What is the MOST LIKELY autonomic symptom this gentleman may suffer?A. Erectile dysfunction.
B. Dry mouth.
C. Urinary incontinence.
D. Heat intolerance.
A 62-year-old man with a history of amyloid light-chain (AL) amyloidosis (after two cycles of Velcade® [bortezomib]). He states that he had numbness in his feet for years, but it worsened VLJQL¿FDQWO DIWHUKLVH[SRVXUHWR9HOFDGHŠ+HDOVRFRPSODLQVof numbness on his left thigh and poor balance. His examination VKRZVQRUPDOUHÀH[HVLQKLVDUPVSDWHOODVDEVHQWDWWKHDQNOHVStrength is 5/5 proximally and distally. Vibration is diminished at the toes. Pinprick sensation is decreased to the knees, and it is also decreased over the left lateral thigh. Nerve conduction studies (NCSs) show the following: Motor Nerve Conduction Studies
Nerve and site
Latency (ms)
Amplitude (mV) Segment
Temperature
difference (ms) (mm)
Ulnar (right)Wrist minimi (manus) – wrist Wrist – below elbow Below elbow –above 31.4 Peroneal (right)Ankle Extensor digitorum Fibula (head) – latency (ms)
latency (ms)
NEUROMUSCULAR VIGNETTES Sensory Nerve Conduction Studies
Nerve and site
latency latency ȝ9
Radial (right)Forearm anatomical snuff box–forearm 31.6 Ulnar (right)Wrist Sural (right)Lower Leg Sural (right)Lower leg Radial (right)Forearm Anatomical snuff box-forearm 30.7 Spontaneous activity Maximum patient effort Ins Activity Fibs hallucisFirst dorsal $PS DPSOLWXGH 'XU GXUDWLRQ &RQ¿J FRQ¿JXUDWLRQ 3RO 3RO SKDVLF )DVF IDVLFXODWLRQ )LEV ¿EULOODWLRQ )U "" ,QV LQVHUWLRQDO 08$3 PRWRU XQLWaction potential, SI=Slightly 3A. What in the patient's history suggests chemotherapy-induced This patient presents with intermittent numbness, tingling, and A. History of AL amyloidosis.
EXUQLQJ LQ WKH ULJKW WKXPE DQG LQGH[ PLGGOH DQG ULQJ ¿QJHUV B. Time course of neuropathy linked to chemotherapy She experienced no other symptoms and had normal sensation, including two-point discrimination and normal abductor pollicis C. Lack of progression once chemotherapy agent brevis strength and needle EMG at the time of the electrodiagnostic (EDX) study. The following recording was obtained.
D. None of the above.
3B. What is the MOST COMMON presentation of chemotherapy induced neuropathy?A. Demyelinating neuropathy.
B. Axonal neuropathy, predominantly motor.
C. Axonal neuropathy, predominantly sensory.
D. $[RQDO QHXURSDWK  VPDOO ¿EHU V PSWRPV SUHGRPLQDQW 3C. What is the treatment for chemotherapy-induced neuropathy? 4A. Which of the following statements concerning this patient's A. Treatment of neuropathic pain.
diagnosis of carpal tunnel syndrome (CTS) is TRUE? B. Increasing the dosage or exposure to chemotherapeutic A. ,W LV PLOG EHFDXVH RI WKH DEVHQFH RI QHHGOH ¿QGLQJV agent suspected.
B. It is severe because of prolonged latency.
C. Decreasing the dosage or stopping the agent.
C. It is mild because of intermittent symptoms.
D. No change in treatment; it is just an expected side effect D. ,WV VHYHULW XQNQRZQ EHFDXVH RI LQVXI¿FLHQW 1&6V of treatment.
E. ,WV VHYHULW XQNQRZQ EHFDXVH RI LQVXI¿FLHQW KLVWRU  NEUROMUSCULAR UPDATE II: MIND THE GAP! BETWEEN THEORY AND PRACTICE An additional recording was performed and the following VIGNETTE SIX
information was obtained: 7KLV SDWLHQW LV D  HDUROG PDQ ZKR KDG GLI¿FXOW DEGXFWLQJDQG ÀH[LQJ KLV VKRXOGHUV ZKLVWOLQJ DQG GULQNLQJ IURP D VWUDZHis mother had similar problems.
(A video will be presented during the live course.) Questions
6A. This patient, seen in the previous video clips, exhibits a
Beevor's sign. The patient's symptoms and signs are consistent with a diagnosis of which of the following?A. T 10 spinal cord injury.
B. Kennedy disease.
C. Myotonic dystrophy.
D. Limb girdle dystrophy.
E. Fascioscapulohumeral muscular dystrophy.
6B. If the biceps is strong, the perceived weakness of elbow 4B. Which of the following statements concerning this patient's ÀH[LRQ LV GXH WR D ZHDN diagnosis of CTS is TRUE? A. Posterior deltoid.
A. ,W LV PLOG EHFDXVH RI WKH DEVHQFH RI QHHGOH ¿QGLQJV B. Latissimus dorsi.
B. It is mild because of intermittent symptoms.
C. Rhomboids.
C. It is severe based on the history, physical examination, D. Upper trapezius.
and EDX studies.
E. Serratus anterior.
D. 7KH ZULVWWRLQGH[ ¿QJHU VHQVRU QHUYH DFWLRQ SRWHQWLDO (SNAP) represents conduction block.
E. 7KH ZULVWWRLQGH[ ¿QJHU 61$3 LV GXH WR SDWKRORJLFDO In 1953, a 15-year-old boy developed sudden onset of total, SDLQOHVV ÀDFFLG SDUDO VLV RI WKH SUR[LPDO ULJKW DUP DQG DOPRVW 4C. Upon further questioning, the patient volunteers that she total paralysis of the distal right limb. His sensation was normal.
awakens 2-3 times every other night with symptoms. Which of the following statements concerning this patient's diagnosis of CTS is TRUE? 7A. Which of the following is the MOST LIKELY diagnosis? A. ,W LV PLOG EHFDXVH RI WKH DEVHQFH RI QHHGOH ¿QGLQJV A. Traumatic plexopathy.
B. It is severe because of conduction block at the wrist.
B. Neuralgic amyotrophy.
C. It is mild because of intermittent symptoms.
C. Functional paralysis.
D. ,WV VHYHULW XQNQRZQ EHFDXVH RI LQVXI¿FLHQW 1&6V D. Guillain–Barré syndrome.
E. Based on the history, physical examination, and EDX studies, it is severe because of the symptoms.
7B. The boy had serratus weakness. Which is the BEST way to test for this?A. Having him push against a wall.
This patient had a radical neck dissection. After the surgery he B. Having him perform a push-up.
KDG GLI¿FXOW DEGXFWLQJ KLV VKRXOGHU LQ WKH FRURQDO SODQH C. Accepting that the muscle cannot be tested.
(A video will be presented during the live course.) D. None of the above.
5A. The weak shoulder abduction seen in the video is due to weakness of which of the following muscles? A 65-year-old man presents with a chief complaint of increasing A. Supraspinatus.
right arm weakness over the last year. His past medical history is B. Rhomboids.
unremarkable. His family history is notable for an older brother C. Posterior deltoid.
with dementia with onset in his late 60s.
D. Pectoralis major.
E. Trapezius.
Examination reveals fasciculations in the tongue, both arms, and right thigh. There is atrophy of the right lateral hand as well as to a milder degree the whole right arm. Tone is increased in all limbs. 7KHUH LV ZHDNQHVV RI WKH DWURSKLHG PXVFOHV 'HHS WHQGRQ UHÀH[HVare increased in the arms and legs as as are the jaw jerk and gag response. NCSs are notable only for the compound muscle action NEUROMUSCULAR VIGNETTES potentials being smaller in the right arm versus the left. Needle EMG reveals widespread acute and chronic denervation changes in three limbs and bulbar muscles.
and extension. She has some neck discomfort during longer car rides. What would be the MOST APPROPRIATE initial 8A. Which genetic marker of amyotrophic lateral sclerosis (ALS) A. Soft neck collar.
is MOST LIKELY to be positive in this patient? B. Wire brace (e.g., Headmaster).
A. Superoxide dismutase 1 (SOD1) (other than A4V).
C. Rigid plastic collar (e.g., Philadelphia, Aspen®).
D. Wheelchair support with strap.
C. TDP-43.
D. SOD1 A4V.
9B. Her neck weakness is now moderate and particularly troubling for holding her head up from her chest. What would be the 8B. Which of the following associations between gene and MOST APPROPRIATE brace to use during the day? phenotype is the MOST UNLIKELY? A. Soft neck collar.
A. SOD1—ALS or progressive muscular atrophy (PMA).
B. Wire brace (e.g., Headmaster).
B. C9orf72—ALS or frontotemporal degeneration (FTD).
C. Rigid plastic collar (e.g., Philadelphia, Aspen®).
C. Valosin-containing protein (VCP)—ALS, FTD, or D. Wheelchair neck support with strap or arms.
inclusion body myositis (IBM).
D. ALSIN—adult ALS or PMA.
9C. Her neck weakness is now severe as well as her limb weakness.
She uses a motorized wheelchair most of the day and 8C. Which of the following statements about the genetics of ALS some nights. She often feels a bit claustrophobic with all her equipment on and has had some skin breakdown A. Most cases of familial ALS are dominantly inherited.
from her current neck brace. What would be the MOST B. Several genes can cause both ALS and FTD and the APPROPRIATE brace for her to use? GLVHDVHV DUH QRZDGD V RIWHQ UHJDUGHG DV UHÀHFWLQJ A. Soft neck collar.
different phenotypes of the same underlying pathology.
B. Wire brace (e.g., Headmaster).
C. Rigid plastic collar (e.g., Philadelphia, Aspen®).
gene in apparently sporadic ALS.
D. Wheelchair support with strap.
D. C9orf72 hexa repeat expansions are seen in 5-21% of apparently sporadic ALS cases.
VIGNETTE TEN
A 45-year-old man presents with 4 years of gradually worsening muscle cramps and fasciculations. His symptoms started in the A patient with upper limb onset ALS is developing increasing calf muscles, but are now affecting most if not all muscles. He problems with controlling her head. As she is cared for her until has not noted any muscle atrophy, but he has lost weight and is no her death, different neck braces are sequentially used to help this longer overweight. He has no sensory or autonomic complaints.
troubling symptom (see below). At each of the following stages of His general examination is unremarkable. He appears to be ¿W DQG KHDOWK  +LV QHXURORJLF H[DPLQDWLRQ LV QRWDEOH RQO  IRUfrequent—almost continuous—fasciculations in many muscle groups. He experiences one calf cramp during strength exam, but there is clearly no weakness or atrophy.
The EDX study is normal with respect to NCSs; repetitive nerve conduction was without decrement or increment in the ulnar QHUYH 1HHGOH (0* VKRZHG QR ¿EULOODWLRQV RU SRVLWLYH VKDUSwaves. There were several fasciculations seen in three limbs affecting two or more muscles supplied by different nerves. The fasciculations did not appear complex, but rather seemed to have a normal motor unit morphology. The patient experienced a couple of cramps during the study, one in the right gastrocnemius that was able to be recorded. The cramp discharge had a typical DSSHDUDQFH¿ULQJLUUHJXODUO DW+] NEUROMUSCULAR UPDATE II: MIND THE GAP! BETWEEN THEORY AND PRACTICE Examination shows the following: 10A. What is TRUE regarding this patient's risk of ALS? ‡ Height: 154 cm. Weight: A little over 40 kg.
A. He does not have ALS, but he may have a slightly increased lifetime risk.
‡ Mental status: Normal.
B. He has ALS by the Lambert criteria.
C. He does not have ALS and will never get it.
‡ Cranial nerves: Pupils are slightly irregular, going from 6 to 3. Eye adduction is reduced 60%, and up gaze is severely 10B. Which of the following conditions have been reported to reduced with little movement. She has marked bilateral present as a cramp and fasciculations syndrome? ptosis. Normal facial sensation. Moderate bilateral facial weakness including all aspects of the face. Normal hearing. B. Becker dystrophy. Normal palatal movement. Sternocleidomastoids are well C. Voltage-gated potassium antibodies. formed but weak, 4/5. Trapezii are 5/5. There is a slight extra D. Acetylcholine receptor antibodies. furrow on the right side of the tongue.
E. All of the above.
‡ Motor: Generally reduced bulk, but no focal atrophy. No 10C. According to the Cochrane Review there is only one fasciculations. Tone is normal.
medication with proven effect against muscle cramps, stating that "There is moderate quality evidence that _ ‡ 6WUHQJWK 1HFN ÀH[LRQ DQG H[WHQVLRQ  6KRXOGHU DEGXFWLRQ VLJQL¿FDQWO UHGXFHV FUDPS IUHTXHQF  LQWHQVLW DQG FUDPS  ZLWK PRUH GLVWDO PXVFOHV EHLQJ VWURQJ +LS ÀH[LRQ days more than placebo." To which medication does the  .QHH H[WHQVLRQÀH[LRQ IRRW SODQWDU DQG GRUVLÀH[LRQ A. Carbamazepine.
B. Quinine.
‡ 5HÀH[HV %LFHSV EUDFKLRUDGLDOLV WULFHSV  TXDGULFHSV DQG C. Gabapentin.
Achilles 2+. Babinski sign absent.
D. Magnesium.
‡ Sensory: Intact to light pinprick and vibration.
6LQJOH ¿EHU (0* ZDV DWWHPSWHG EXW WKH SDWLHQW ZDV XQDEOH A 25-year-old woman presents for 10 years of progressive eyelid to tolerate the examination. Repetitive nerve stimulation was drooping. She denies frank double vision, but notes some blurring normal. Creatine kinase was normal at 68. The anti-acetylcholine of vision on lateral gaze. She has been experiencing worsening UHFHSWRU DQWLERG DQG PXVFOHVSHFL¿F W URVLQH NLQDVH DQWLERG fatigue over the last 3 years. Initially, she attributed this to the tests were negative/normal. Thyroid studies were normal.
birth of her daughter and subsequent sleep disturbances, but despite her daughter sleeping through the night her fatigue is worsening. Her past medical history is unremarkable; she has A muscle biopsy reveals the following on trichrome staining: had cholecystectomy and ptosis surgery. She is the youngest of six healthy siblings and her parents are alive and without health problems. Her review of systems is notable for palpitations. She is on birth control pills and supplements.
11A. What can be observed? A. Abnormal cytoplasmic glycogen accumulations.
B. Red rimmed vacuoles.
C. 5DJJHG UHG ¿EHUVD. Increased perimysial connective tissue.
NEUROMUSCULAR VIGNETTES 11B. Which of the following statements concerning the genetics of this condition is the MOST ACCURATE? A. It is always caused by a mitochondrial mutation, either a duplication, deletion, or point mutation.
B. It is most often recessively inherited.
C. Mutations in the mitochondrial and somatic DNA can cause this phenotype.
D. Mitochondrial diseases are always homoplastic (i.e., the mitochondria in one cell are genetically identical).
11C. In addition to muscle, many other tissues are often affected by mitochondrial cytopathies. Some of these multisystem diseases are referred to by an acronym. Which of the following is NOT an acronym used to refer to a mitochondrial cytopathy syndrome? A. CPEO.
B. MELAS.
C. MFN2.
D. KSS.

Source: https://webportal.aanem.org/Resources/Files/Downloads/products/13CI.pdf

stadtlinz.eu

Radionuclide therapy beyond radioiodine Wiener Medizinische Wochenschrift ISSN 0043-5341Wien Med WochenschrDOI 10.1007/s10354-012-0128-6 Your article is protected by copyright and all rights are held exclusively by Springer- Verlag Wien. This e-offprint is for personal use only and shall not be self-archived in

Slg august.indt

‘Bio Piracy' — A Discussion of Some Important Cases The article discusses the issue of large pharmaceutical companies patenting bio-resources that have been traditionally used by the indigenous people of a land. Bio piracy is the appropriation of the to traditional knowledge,2 some of the an ancient Sanskrit text and a paper knowledge and genetic resources of